Category Archives: Lớp 9

Suy luận phản chứng (phần 2)

Phép phản chứng trong toán học còn được gọi là phương pháp chứng minh bằng mâu thuẫn. Nếu ta muốn chứng minh kết luận của bài toán là đúng thì cần phải chứng minh điều ngược lại với giả thiết là sai. Sau đây ta xét một vài ví dụ áp dụng suy luận này, dành cho các bạn hs lớp 8, 9.

1/ Ví dụ:

Ví dụ 1. 

Chứng minh rằng $\sqrt{2}$ là một số vô tỷ.

Lời giải

Giả sử $\sqrt{2}$ là số hữu tỉ. Khi đó tồn tại $a,b\in \mathbb{N}^*$ sao cho $\sqrt{2}= \dfrac{a}{b}$ với $(a,b)=1$

Ta có: $(\sqrt{2})^2=\left(\dfrac{a}{b}\right)^{2}$ hay $a^{2}=2 b^{2}\quad (1)$

Suy ra a là số chẵn, ta có: $\mathrm{a}=2 \mathrm{c}$ với $c\in Z$

Thay $\mathrm{a}=2 \mathrm{c}$ vào (1) ta được: $(2 c)^{2}=2 b^{2}$ hay $b^{2}=2 c^{2}$

Do đó, b là số chẵn

Hai số a và $b$ đều số chẵn $\Rightarrow$ Mâu thuẫn với $(1)$

Vậy $\sqrt{2}$ là số vô tỉ.

Ví dụ 2. 

Chứng minh rằng tổng của một số hữu tỷ và một số vô tỷ là số vô tỷ.

Lời giải

Giả sử tổng của số hữu tỉ a vs số vô tỉ b là số hữu tỉ c, ta có: $\mathrm{b}=\mathrm{c}-\mathrm{a}$

Mà hiệu của 2 số hữu tỉ phải là số hữu tỉ nên $b$ là số hữu tỉ

$\Rightarrow$ Mâu thuẫn vs giả thiết

Vậy tổng của 1 số hữu tỉ với 1 số vô tỉ là 1 số vô tỉ.

Ví dụ 3. (Nguyên lý Dirichlet)

Có $nk + 1$ viên bi, bỏ vào trong $k$ cái hộp. Chứng minh rằng có ít nhất một hộp có ít nhất là là $n+1$ viên bi.

Lời giải

Giả sử tất cả các hộp đều chứa số bi không vượt quá $n$ viên, khi đó tổng số bi không vượt quá $nk$, mâu thuẫn. Vậy phải có một hộp chứa nhiều hơn $n$ viên bi $\Rightarrow$ đpcm.

2/ Bài tập

Bài 1. 

Cho 15 số phân biệt thỏa mãn tổng của 8 số bất kì lớn hơn tổng của 7 số còn lại. Chứng minh tất cả các số đã cho đều dương.

Lời giải

Gọi 15 số đã cho là $a_1<a_2<a_3<\cdots <a_{15}$. Ta chỉ cần chứng minh $a_1 > 0$.

Thật vậy, giả sử $a_1 \leq 0$, khi đó $$a_1 + a_2 + \cdots + a_8 \leq a_2 + a_3 + \cdots a_8 < a_9 + \cdots a_{15}$$ (mâu thuẫn).

Vậy điều giả sử là sai, hay  $0<a_1\Rightarrow 15$ số đã cho đều dương.

Bài 2. 

Từ 8 số nguyên dương không lớn hơn 20, chứng minh rằng có thể chọn ra 3 số $x, y, z$ là độ dài 3 cạnh của một tam giác.

Lời giải

Gọi 8 số nguyên dương không lớn hơn 20 là $a_{1}, a_{2}, a_{3}, \ldots, a_{8}$

$$ \text { với } 1 \leq a_{1} \leq a_{2} \leq a_{3} \leq a_{4} \leq \ldots \ldots \leq a_{8} \leq 20 $$

Nhận thấy rằng với ba số nguyên dương $a, b, c$ thỏa mãn $a \geq b \geq c$ và $b+c>a$ thì khi đó $a, b, c$ là độ dài 3 cạnh tam giác.

Giả sử trong các số $a_{1}, a_{2}, a_{3}, a_{4}, \ldots . a_{8}$ không chọn được 3 số nào là độ dài 3 cạnh của tam giác thì ta có:

$$a 3 \geq a 1+a 2 \geq 1+1=2$$

$$a 4 \geq a 2+a 3 \geq 1+2=3$$

$$a 5 \geq a 3+a 4 \geq 2+3=5$$

$$a 6 \geq a 4+a 5 \geq 3+5=8$$

$$a 7 \geq a 5+a 6 \geq 5+8=13$$

$$a 8 \geq a 6+a 7 \geq 13+8=21$$

$\Rightarrow$ Trái với giả thiết

Vậy điều giả sử là sai

$\Rightarrow$ đpcm.

Bài 3. 

Cho tập $B = {1, 2, 3, …, 16}$. Người ta ghi các số của tập B thành một vòng tròn (mỗi số ghi một lần). Hỏi có cách ghi để tổng thỏa:

a/ Tổng của hai số kề nhau bất kì lớn hơn hoặc bằng 17 được không? Tại sao?

b/ Tổng của ba số kề nhau bất kì lớn hơn 24 được không? Tại sao?

Lời giải

a/ Giả sử tồn tại cách ghi thỏa mãn. Khi đó, gọi 2 số kề với 1 là a và b.

Theo giả thiết, ta có:

$\left\{\begin{array}{l} 1 + a \geqslant 17  \\1 + b \geqslant 17  \end{array} \right. \Rightarrow \left\{\begin{array}{l}  a \geqslant 16 \\ b \geqslant 16 \end{array} \right. \Rightarrow$ Mâu thuẫn.

Vậy không tồn tại cách ghi thỏa mãn.

b/ Giả sử tồn tại cách ghi thỏa mãn.

Khi đó, ta tách số 16 ra và chia 15 số còn lại thành 5 bộ 3 số kề nhau. Và tổng của 16 số này phải lớn hơn hoặc bằng: $16+5\cdot 25=141$

Mà $1+2+3+\cdots 16=136 \Rightarrow $ Mâu thuẫn

Vậy không tồn tại cách ghi thỏa mãn.

Bài 4. 

Có thể chia tập $X = \{1, 2, …, 2023\}$ thành hai tập rời nhau sao cho tổng các phần tử thuộc tập này bằng 2 lần tổng các phần tử thuộc tập kia?

Lời giải

Giả sử có thể chia tập $X$ thành hai tập rời nhau $A$ và $B$ sao cho tổng các phần tử thuộc A bằng 2 lần tổng các phần tử thuộc B.

Khi đó, tổng các phần tử của 2 tập hợp này phải chia hết cho 3.

Mà ta có: $1+2+3+\cdots +2023=\dfrac{2023\cdot 2024}{2}=1012\cdot 2023 \not \vdots \ 3 \Rightarrow$ Mâu thuẫn

Vậy không thể chia tập $X$ thành hai tập rời nhau $A$ và $B$ sao cho tổng các phần tử thuộc $A$ bằng 2 lần tổng các phần tử thuộc $B$.

Bài 5. 

Một bảng vuông $8 \times 8$ khuyết các ô vuông ở hai góc đối diện. Hỏi có thể phủ các ô của bảng vuông bằng các hình Domino $1 \times 2$ mà không có quân Domino nào chồng lên nhau được không? Tại sao?

Lời giải

Không có mô tả.

Giả sử có thể phủ các ô của bảng vuông bằng các hình Domino $1 \times 2$ mà không có quân Domino nào chồng lên nhau.

Mỗi quân Domino lát vào bàn cờ luôn chiếm một ô trắng và một ô đen. Do đó, để lát được phần còn lại của bàn cờ thì số ô trắng và số ô đen bằng nhau. Mà số ô màu trắng và số ô màu đen trong phần còn lại của bàn cờ không bằng nhau. Điều này mâu thuẫn.

Vậy không thể lát được phần còn lại của bàn cờ bằng các quân Domino.

TẬP HỢP – TẬP HỢP SỐ

Ví dụ 1.1. Số nguyên $A$ được tạo thành bằng các chữ viết liền nhau các số nguyên dương từ 1 đến 60 theo thứ tự từ nhỏ đến lớn: $A=123 \ldots 585960$.
(a) Hãy chỉ ra cách xóa 100 chữ số của $A$ sao cho số $A_1$ tạo bởi các chữ số còn lại là nhỏ nhất.
(b) Hãy chỉ ra cách xóa 100 chữ số của $A$ sao cho số $A_2$ tạo bởi các chữ số còn lại là lớn nhất.

Hướng dẫn giải

(a) Số $A$ có $9+2.51=111$ chữ số. Sau khi xóa 100 chữ số của $A$ ta còn 11 chữ số.
Ta có: $A=12 \ldots 10 \ldots 20 \ldots 30 \ldots 40 \ldots 50 \ldots 60$ có 6 chữ số 0 .
Để $A_1$ nhỏ nhất ta sẽ xóa sao cho $A_1$ có nhiều số 0 đứng đầu nhất.
Theo phân bố của các số 0 trong $A$ thì số $A_1$ có thể có tối đa 5 chữ số 0 đứng đầu. Còn lại 6 chữ số của $A_1$ sẽ được lấy từ dãy số sau: 51525354555657585960 .
Vậy số $A_1=00000123450$ là số nhỏ nhất cần tìm.
(b) Tương tự lập luận ở câu a)
Ta có: $A=1 \ldots 9 \ldots 19 \ldots 29 \ldots 39 \ldots 49 \ldots 5960$ có 6 chữ số 9 .
Để $A_2$ lớn nhất thì ta sẽ xóa sao cho $A_2$ có nhiều số 9 đứng đầu nhất.
Theo phân bố của các số 9 trong $A$ thì số $A_2$ có thể có tối đa 5 chữ số 9 đứng đầu. Còn lại 6 chữ số của $A_2$ sẽ được lấy từ dãy số sau: 51525354555657585960 .
Vậy số $A_2=99999785960$ là số lớn nhất cần tìm.

Ví dụ 1.2. Cho tập $A=\{1,2,3, \ldots, 9\}$.
(a) Hãy chỉ ra một cách chia tập $A$ thành 3 tập con rời nhau, có số phần tử bằng nhau và tổng các phần tử bằng nhau.
(b) Tìm tất cả cách chia trong câu a.

Hướng dẫn giải

(a) $A_1=\{1,5,9\}, A_2=\{2,6,7\}, A_3=\{3,4,8\}$ là một cách chia thỏa đề bài.
(b) Tổng các phần tử là $1+2+\cdots+9=45$ do đó mỗi tập hợp có tổng là 15 và có 3 phần tử.
Dễ thấy $1,2,3$ không cùng một tập hợp, vì nếu cùng thì phần tử còn lại sẽ lớn hơn hoặc bằng 10 (vô lý).
Giả sử $1 \in A_1, 2 \in A_2, 3 \in A_3$. hai phần tử còn lại của $A_1$ là $a, b$, ta có $a+b=14$, chỉ có thể là 6,8 hoặc 5,9.
Nếu $6,8 \in A_1$, thì hai phần tử thuộc $A_2$ tổng là 13, chỉ có thể là 4,9 .
Khi đó $5,7 \in A_3$. Ta có các kết quả $A_1=\{1,6,8\}, A_2=\{2,4,9\}, A_3=\{3,5,7\}$.
Nếu $5,9 \in A_1$, thì hai phần tử thuộc $A_2$ có tổng 13 là 6,7.
Khi đó $4,8 \in A_3$. Các kết quả là $A_1=\{1,5,9\}, A_2=\{2,6,7\}, A_3=\{3,4,8\}$.

Ví dụ 1.3. Biết rằng:
$$
A=\{1 ; a\}, B=\{a ; b ; 3\}, C=\{2 ; 4 ; c\}, D=\{a ; b ; 4\}, E=\{a ; b ; c ; e\}
$$
và biết $A \subset D ; B \subset E ; C \subset E ; D \subset E$. Tìm các phần tử $a, b, c, e$.

Hướng dẫn giải

Từ $A \subset D$, suy ra $b=1$.
Từ $B \subset E$, thì một trong hai số $c$ hoặc $e$ phải là $3(1)$.
Từ $D \subset E$ thì một trong hai số $c$ hoặc $e$ phải là $4(2)$.
Từ $C \subset E$ và (1),(2) thì $c, e$ không nhận giá trị 2 nên $a=2$ và $e=4$, suy ra $c=3$.
Vậy $a=2, b=1, c=3, e=4$.

Ví dụ 1.4. Tập hợp $M$ chứa 4 số nguyên phân biệt được gọi là tập liên kết nếu với mỗi $x \in M$ thì ít nhất một trong hai số $x-1, x+1$ thuộc $M$. Gọi $U_n$ là số tập con liên kết của tập $\{1,2, \ldots, n\}$.
(a) Tính $U_7$.
(b) Xác định giá trị nhỏ nhất của $n$ sao cho $U_n \geq 2019$.

Hướng dẫn giải

Gọi $a<b<c<d$ là 4 phần tử của một tập liên kết M.
Vì $a-1 \notin M $ nên $a+1 \in M$, suy ra $b=+1$. Vì $d-1 \in M$, suy ra $c=d-1$.
Như vậy một tập liên kết sẽ có dạng $\{a+1, d-1, d\}$, với $\{d-a>2\}$.
(a) Có 10 tập con liên kết của tập $\{1,2,3,4,5,6,7\}$ là
$$
\begin{aligned}
& \{1,2,3,4\},\{1,2,4,5\},\{1,2,5,6\},\{1,2,6,7\}, \
& \{2,3,4,5\},\{2,3,5,6\},\{2,3,6,7\}, \
& \{3,4,5,6\},\{3,4,6,7\},\{4,5,6,7\} .
\end{aligned}
$$
(b) Gọi $D=d-a+1$ là đường kính của tập $\{a, b=a+1, c=d-1, d\}$, hiển nhiên $3<D \leq$ $n-1+1=n$.
Với $D=4$ sẽ có $n-3$ tập liên kết, với $D=5$ sẽ có $n-4$ tập liên kết, …, với $D=n$ sẽ có đúng một tập liên kết. Do đó
$$
U_n=1+2+\ldots+(n-3)=\dfrac{(n-3)(n-2)}{2} .
$$
Do đó $U_n \geq 2019 \Leftrightarrow(n-3)(n-2) \geq 4038$. Như vậy giá trị nhỏ nhất của $n$ là $n=67$.

Ví dụ 1.5. Chứng minh rằng với mọi số dương $m$ thì $\dfrac{2 m}{m^2+5}$ không thể là số nguyên.

Hướng dẫn giải

Ta có $0<\dfrac{2 m}{m^2+5}<1$ nên $\dfrac{2 m}{m^2+5}$ không thể là số nguyên.

Ví dụ 1.6. (Đề tuyển sinh vào lớp 10 chuyên toán trường PTNK năm 2014) Cho 5 số tự nhiên phân biệt sao cho tổng của ba số bất kỳ trong chúng lớn hơn tổng của hai số còn lại.
(a) Chứng minh rằng tất cả 5 số đā cho đều không nhỏ hơn 5 .
(b) Tìm tất cả các bộ gồm 5 số thỏa mãn đề bài mà tồng của chúng nhỏ hơn 40 .

Hướng dẫn giải

(a) Gọi 5 số đó là $a, b, c, d, e$, do các số là phân biệt nên ta có thể giả sử $ad+e$, suy ra $a+b+c \geq d+e+1$. Suy ra $a \geq d+e+1-b-c$.
Mặt khác, do $b, c, d, e$ là số tự nhiên nên từ $d>c>b$ ta có $d \geq c+1 \geq b+2$, suy ra $d-b \geq 2$. $e>d>c$, suy ra $e-c \geq 2$.
Do đó $a \geq(d-b)+(e-c)+1 \geq 5$. Suy ra $b, c, d, e>5$.
Vậy các số đều không nhỏ hơn 5.
(b) Nếu $a \geq 6$, suy ra $b \geq 7, c \geq 8, d \geq 9, e \geq 10$, suy ra $a+b+c+d+e \geq 40$ ( vô lý),
suy ra $a<6$.
Theo câu a ta có $a=5$. Khi đó $b+c+5 \geq d+e+1$, suy ra $b+c \geq d+e-4$.
Mà $d-2 \geq b, e-2 \geq c$, suy ra $d+e-4 \geq b+c$. Do đó $b=d-2, c=e-2$.
Khi đó $a+b+c+d+e=5+2 b+2 c+4<40$. Suy ra $b+c<\dfrac{31}{2}$. Suy ra $b \geq 7$.
Từ đó ta có $b=6, b=7$.
Nếu $b=6$ ta có $d=8, c=8, e=10$. Ta có bộ $(5,6,7,8,9)$
Nếu $b=7, d=9, c=8, e=10$.
Ta có bộ $(5,7,8,9,10)$. Vậy có hai bộ số thỏa đề bài là $(5,6,7,8,9)$ và $(5,7,8,9,10)$.

Ví dụ 1.7. Trong một buôn của người dân tộc, cư dân có thể nói được tiếng dân tộc, có thể nói được tiếng Kinh hoặc nói được cả hai thứ tiếng. Kết quả của một đợt điều tra cơ bản cho biết:
Có 912 người nói tiếng dân tộc,
Có 653 người nói tiếng Kinh,
Có 435 người nói được cả hai thứ tiếng.
Hỏi buôn làng có bao nhiêu cư dân ?

Hướng dẫn giải

Gọi $A$ là tập các người các người nói tiếng dân tộc, ta có $|A|=912, B$ là tập các người nói tiếng Kinh, ta có $|B|=653$. Khi đó $|A \cap B|=435$.
$A \cup B$ là tập các người dân trong buông.
Ta có
$$
|A \cup B|=|A|+|B|-|A \cap B|=912+653-435=1130
$$

Bài 1.1. Viết các số từ 1 đến 9 vào một bảng vuông $3 \times 3$, mỗi số viết một lần, sao cho tồng số ở mỗi dòng, mỗi cột và hai đường chéo đều được số chia hết cho 9 .
(a) Chỉ ra một cách viết thỏa đề bài.
(b) Với cách viết thỏa đề bài thì ô chính giữa có thể là các số nào? Tại sao?

Hướng dẫn giải

(a)
(b) Giả sử ta có bảng sau thỏa đề bài

Ta có $a+e+k, c+e+g, d+e+f, b+e+h$ chia hết cho 9 .

$$
a+e+k+c+e+g+d+e+f+b+e+h=3 e+a+b+c+d+e+f+g+h+k=3 e+45
$$
nên $3 e+45$ chia hết cho 9 , do dó $e$ chia hết cho 3 , vậy $e \in\{3,6,9\}$.

Bài 1.2. Tích của $n$ số nguyên bằng 1 và tổng của chúng bằng 0 . Chứng minh rằng $n$ là một số chia hết cho 4 .

Hướng dẫn giải

Gọi $n$ số đó là $a_1, a_2, \cdots, a_n$. Ta có
$$
a_1+a_2+\cdots+a_n=0
$$

$$
a_1 \cdot a_2 \cdots a_n=1
$$
nên các số $a_i \in\{-1 ; 1\}$, mà tổng bằng 0 nên số các số 1 bằng số các số -1 , do đó $n$ chẵn, đặt $n=2 k$, khi đó
$$
1=a_1 \cdot a_2 \cdots a_n=(-1)^k
$$
Do đó $k$ cũng chẵn, suy ra $n$ chia hết cho 4.

Bài 1.3. Tập hợp $\mathrm{A}$ bao gồm các số tự nhiên thỏa các điều kiện sau:
(a) $1 \in A$;
(b) Nếu $n \in A$ thì $2 n+1 \in A$;
(c) Nếu $3 n+1 \in A$ thì $n \in A$;
Vậy 8 có thuộc $A$ không ?

Hướng dẫn giải

$\{1,3,7,15,31,63,127\} \in A$, và $\{42,85,171,343,114,229,76,25,8\} \in A$

Bài 1.4. Giả sử $x, y, z, t$ là bốn số khác nhau và là các phần tử của tập hợp
$$
A=\{1 ; 2 ; 3 ; 4\} .
$$
Tìm $x, y, z, t$ với các giả thiết:
Nếu $x \neq 1$ thì $z \neq 2$;
Nếu $t=2$ thì $y \neq 1$;
Nếu $y=2$ hoặc $y=3$ thì $x=1$;
Nếu $y \neq 3$ thì $z=4$;
Nếu $t \neq 1$ thì $y=1$.

Hướng dẫn giải

Bài 1.5. Một nhóm 6 học sinh làm bài kiểm tra môn toán được điểm là số tự nhiên từ 1 đến 10 . Hai bạn được gọi là bạn tốt nếu điểm trung bình của 2 bạn đó lớn điểm trung bình của 6 bạn.
(a) Có thể chia 6 bạn thành 3 cặp bạn tốt được không? Tại sao?
(b) Nếu số điểm của 6 bạn là khác nhau, chứng minh rằng có 2 bạn có số điểm hơn kém nhau là 1 .

Hướng dẫn giải

Gọi số điểm các bạn lằn lượt là $a_1, a_2, a_3, a_4, a_5, a_6$, và $a_i \in\{1,2,3,4,5,6,7,8,9,10\}$.
Đặt $s=a_1+a_2+a_3+a_4+a_5+a_6$
(a) Giả sử chia được thành 3 cặp bạn tốt, giả sử là các cặp $a_1, a_2 ; a_3, a_4$ và $a_5, a_6$ ta có
$$
\dfrac{a_1+a_2}{2}>\dfrac{s}{6}, \dfrac{a_3+a_4}{2}>\dfrac{s}{6}, \dfrac{a_5+a_6}{2}>\dfrac{s}{6}
$$
Suy ra
$$
\dfrac{a_1+a_2+a_3+a_4+a_5+a_6}{2}>\dfrac{s}{2}
$$

Điều này mâu thuẫn.
(b) Giả sử không có bạn nào hơn kém nhau là 1 , thì giả sử $a_1<a_2<a_3<a_4<a_5<a_6$ Suy ra $a_2 \geq 3, a_3 \geq 5, \cdots, a_6 \geq 11$, vô lí.

Bài 1.6. Trong kỳ thi tốt nghiệp THPT ở một trường, kết quả số thí sinh đạt danh hiệu xuất sắc nhu sau:
Về môn Toán: 48 thí sinh,
Về Toán hoặc Văn: 76 thí sinh,
Về Vật lí: 37 thí sinh,
Về Văn: 42 thí sinh,
Về Vật lí hoặc Văn: 66 thí sinh,
Về Toán hoặc Vật lí: 75 thí sinh,
Về cả ba môn: 4 thí sinh.
Vậy có bao nhiêu học sinh chỉ nhận được danh hiệu xuất sắc về:
(a) 1 môn ?
(b) 2 môn?
(c) Ít nhất 1 môn?

Hướng dẫn giải

Sử dụng biểu đồ Venn. Kí hiệu $A, B, C$ là tập hợp các học sinh đạt danh hiệu xuất sắc tương ứng với các môn Toán, Vật lí hoặc Văn. Các tập hợp này, theo giả thiết thì có 48,37 và 42 phần tử. Giao của ba tập hợp này có 3 phần tử. Kí hiệu qua $\mathrm{a}, \mathrm{b}, \mathrm{c}, \mathrm{x}, \mathrm{y}, \mathrm{z}$ là số các thí sinh đạt danh hiệu xuất sắc.

Theo 1,2 hoặc 3 môn. Dựa vào biểu đồ Venn ta lập được các phương trình:
$$
\left\{\begin{array}{l}
a+x+y=44 \\\
b+x+z=33 \\\
a+b+x+y+z=71 \\\
a+c+x+y+z=72 \\\\
b+c+x+y+z=62
\end{array}\right.
$$
Ta có được một hệ 6 phương trình với 6 ần, nhưng diều mà ta cần biết không phải là các giá trị ẩn $\mathrm{a}, \mathrm{b}, \mathrm{c}, \mathrm{x}, \mathrm{y}, \mathrm{z}$ mà là các tổng $\mathrm{a}+\mathrm{b}+\mathrm{c}, \mathrm{x}+\mathrm{y}+\mathrm{z}$.
Muốn vậy, ta cộng ba phương trình đầu của hệ và sau đó cộng ba phương trình sau của hệ với nhau và được:
$$
\left\{\begin{array}{l}
a+b+c+2(x+y+z)=115 \\\
2(a+b+c)+3(x+y+z)=205
\end{array}\right.
$$
Xem hệ này như là một hệ phương trình hai ẩn, ta tính được:

$$
\begin{aligned}
& a+b+c=65 \
& x+y+z=25
\end{aligned}
$$

Đáp số: 65 thí sinh đạt danh hiệu xuất sắc 1 môn, 25 thí sinh đạt danh hiệu xuất sắc 2 môn, 94 thí sinh đạt danh hiệu xuất sắc ít nhất 1 môn.

Bài 1.7. Một số $m$ được gọi là số ma thuật nếu tổng các chữ số của nó bằng tích các chữ số của nó. Ví dụ số 213 ta có $2+1+3=2 \times 1 \times 3$.
(a) Chứng minh rằng có số ma thuật có $1,2,3,4,5$ chữ số.
(b) Có số ma thuật có 6 chữ số hay không? Tại sao?
(c) Chứng minh rằng có số ma thuật có 2037 chữ số.

Hướng dẫn giải

(a) Các số ma thuật có $1,2,3,4,5$ chữ số là: $1,22,123,4211,52111$.
(b) Số ma thuật có 6 chữ số: 621111
(c) $22222222222111 \ldots .1,11$ chữ số 2 và 2025 chữ số 1 .

Bài 1.8. Có thể viết các số tự nhiên từ 1 đến 16 thành
(a) một đường thẳng
(b) một đường tròn
sao cho tồng hai số liên tiếp là bình phương của một số tự nhiên dược không? Tại sao

Hướng dẫn giải

(a) $8,1,15,10,6,3,13,12,4,5,11,14,2,7,9,16$.
(b) Giả sử tồn tại cách ghi thỏa đề bài, ta xét hai số kề bên số 8 , gọi là $a, b$ thì $8+a, 8+b$ đều là số chính phương, suy ra $a=b=1$, vô lí. Vậy không tồn tại cách ghi thỏa đề bài.

Bài 1.9. Cho $A$ là tập con của tập các số hữu tỷ dương thỏa mãn các điều kiện sau:
$1 \in A$
Nếu $x \in A$ thì $1+x \in A$
Nếu $x \in A$ thì $\dfrac{1}{x} \in A$

Hướng dẫn giải

(c) $\dfrac{13}{5}=2+\dfrac{3}{5}$.
Ta có $\dfrac{3}{5}=\dfrac{1}{1+\dfrac{2}{3}} \dfrac{3}{2} \in A \Rightarrow \dfrac{2}{3} \in A \Rightarrow \dfrac{5}{3}=1+\dfrac{2}{3} \in A$, do đó $\dfrac{3}{5} \in A$, hơn nữa $2 \in A$, suy ra $\dfrac{13}{5}=2+\dfrac{3}{5} \in A$.

Bài 1.10. Trên bảng có ghi các số tự nhiên từ 1 đến $n$. Cứ mỗi lần một học sinh xóa đi hai số và thay bằng tổng hoặc hiệu của hai số đó.
(a) Cho $n=8$ hỏi sau 7 lần có thể số trên bảng còn lại số 0 dược không?
(b) Câu hỏi tương tự với $n=9$.

Hướng dẫn giải

(a) Câu trả lời là thực hiện được, ta làm như sau:
$1,2,3,4,5,6,7,8$
$1,2,3,4,5,6,1$
$1,2,3,4,1,1$
$1,2,1,1,1$
$1,1,1,1$,
$1,1,0$
$0,0$
$0$
(b) Câu trả lời là không, vì mổi lần thay đổi thì tổng các số còn lại tính chẵn lẻ khồng đổi, tổng lúc đầu là $1+2+\cdots+9=45$ nên sau một số lần thay đổi thì số còn lại phải là số lẻ, không thể bằng 0 .

Bài 1.11. Có bao nhiêu cách viết số 1 thành tồng của 3 phân số mà mỗi phân số có tử số bằng 1 và mẫu số là một số tự nhiên? Tại sao?

Hướng dẫn giải

$$
1=\dfrac{1}{6}+\dfrac{1}{3}+\dfrac{1}{2}=\dfrac{1}{4}+\dfrac{1}{4}+\dfrac{1}{2}=\dfrac{1}{3}+\dfrac{1}{3}+\dfrac{1}{3}
$$

Bài 1.12. Chứng minh rằng giữa hai số hữu tỉ phân biệt luôn có một số hữu tỉ.

Hướng dẫn giải

Cho $a, b \in \mathbb{Q}, a<b$. Xét $c=\frac{a+b}{2}$ ta có $a<c<b$ và $c \in \mathbb{Q}$.

Bài 1.13. Gọi $S$ là tập hợp các số tự nhiên có thể viết thành tổng bình phương của hai số tự nhiên khác, ví dụ $5=1^2+2^2$ thì $5 \in S$. Chứng minh rằng nếu $x, y \in S$ thì $x y \in S$.

Hướng dẫn giải

Cho $a, b \in S$ ta có $a=x^2+y^2, b=z^2+t^2$, khi đó
$$
a b=\left(x^2+y^2\right)\left(z^2+t^2\right)=x^2 z^2+y^2 t^2+x^2 t^2+y^2 z^2=(x z+t y)^2+(x z-t y)^2
$$
Do đó $a b \in S$.

Bài 1.14. Cho $a, b$ là các số nguyên dương phân biệt, chứng minh rằng 1 không là nghiệm của phương trình $x^2-2(a+b) x+a b+2=0$.

Hướng dẫn giải

Giả sử 1 là nghiệm của phương trình ta có
$$
1^2-2(a+b) 1+a b+2=0 \Leftrightarrow a b-2 a-2 b+3=0 \Leftrightarrow(a-2)(b-2)=1
$$
Do $a, b$ là các số nguyên dương nên $a=1, b=1$ hoặc $a=3, b=3$ mâu thuẫn vì $a \neq b$.

Bài 1.15. Cho các số $a_1, a_2, \cdots, a_6$ thỏa $-\dfrac{1}{2} \leq a_i \leq \dfrac{1}{2}$ và tổng của 5 số bất kì là một số nguyên. Chứng minh rằng 6 số này bằng nhau.

Hướng dẫn giải

Đặt $S=a_1+a_2+\cdots a_6$, ta có $S \in \mathbb{Z}$
Ta có $S-a_i \in \mathbb{Z}$ với mọi $i$.
Giả sử có hai số $a_1 \neq a_2$ ta có $S-a_1-\left(S-a_2\right) \in \mathbb{Z} \Rightarrow a_2-a_1 \in \mathbb{Z}$, suy ra $a_1, a_2 \in\{\dfrac{1}{2},-\dfrac{1}{2}\}$, do $a_1 \neq a_2$ nên $a_1=\dfrac{1}{2}, a_2=-\dfrac{1}{2}$ hoặc $a_1=\dfrac{-1}{2}, a_2=\dfrac{1}{2}$.
Tương tự xét cặp số giữa $a_1$ với các số $a_3,a_4, a_5, a_6$ ta có cũng có các số còn lại thuộc $\{\dfrac{1}{2}, \dfrac{-1}{2}\}$, do đó tổng 5 số lúc này không thể là số nguyên.

Một số bài toán về đường cao và trực tâm

Trong chương trình hình học chuyên toán dành cho lớp 9, có nhiều bài toán liên quan đến các đường cao và trực tâm tam giác, hôm nay chúng ta sẽ tìm hiểu một số tính chất và bài tập như thế.

Bài 1. Cho tam giác $ABC$ nội tiếp đường tròn $(O)$, các đường cao $AD, BE, CF$ cắt nhau tại $H$. $M$ là trung điểm của $BC$. Vẽ đường kính $AK$.

a) Chứng minh $H, M, K$ thẳng hàng;

b) Chứng minh $AH = 2 \cdot OM$.

c) Gọi $G$ là trọng tâm tam giác $ABC$. Chứng minh $H, G, O$ thẳng hàng và $GH = 2 \cdot OG$.

Hướng dẫn - Gợi ý

(a) Ta có $\angle A B K=\angle A C K=90^{\circ}$, từ đó $B K / / C H, C K / / B H$, do đó tứ giác $B H C K$ là hình bình hành.
Hơn nữa $M$ là trung điểm $B C$ nên cũng là trung điểm $H K$.
(b) Tam giác $A H K$ thì $O, M$ lần lượt là trung điểm $A K, H K$ nên $O M$ là đường trung bình, do đó $A H=2 \cdot O M$.
(c) Tam giác $A B C$ có $A M$ là trung tuyến nên với $G$ là trọng tâm thì ta có $A G=\frac{2}{3} A M$. Hơn nữa tam giác $A H K$ có $A M$ là trung tuyến và $G$ thuộc $A M$ và $A G=\frac{2}{3} A M$ nên $G$ cũng là trọng tâm tam giác $A H K$; Mặt khác có $O$ là trung điểm của $A K$ nên $G$ thuộc đoạn $H O$ và $G H=2 \cdot G O$.

Bài 2. Cho tam giác $A B C$ nhọn nội tiếp đường tròn $(O)$. Các đường cao $A D, B E, C F$ cắt nhau tại $H$. $A O$ căt $E F$ tại $K$ và $(O)$ tại $L$.
a) Chứng minh $\angle B A H=\angle C A O$ và $\angle A O \perp E F$.
b) $C F, B E$ cắt $(O)$ tại $Q, P$. Chứng minh $A P=A Q=A H$.
c) Tính $\angle A$ nếu $B, H, O, C$ cùng thuộc một đường tròn. Khi đó tính $\angle O H C$.

Hướng dẫn - Gợi ý

(a) Ta có $\angle B A D=90^{\circ}-\angle A B D, \angle C A K=90^{\circ}-\angle A K C$;
Hơn nữa $\angle A B D=\angle A K C$, suy ra $\angle B A D=\angle C A K$.
Gọi $L$ là giao điểm của $A K$ và $E F$.
Tứ giác $B F E C$ nội tiếp, suy ra $\angle A E F=\angle A B C=\angle A K C$, do đó tứ giác $E L K C$ nội tiếp.
Từ đó $\angle A L E=\angle A C K=90^{\circ}$ hay $A O \perp E F$.

(b) Ta có $\angle A P H=\angle A C B=\angle A H P $ suy ra $ A H=A P$. Tương tự thì $A H=A Q$

(c) Ta có $\angle B H C=\angle E H F=180^{\circ}-\angle B A C$; Và $\angle B O C=2 \angle B A C$; Khi đó $B H O C$ nội tiếp khi và chỉ khi $\angle B H C=\angle B O C$ hay $180^{\circ}-\angle B A C=2 \angle B A C$, tính ra $\angle B A C=60^{\circ}$.
Khi đó $\angle O H C=\angle O B C=30^{\circ}$.

Bài 3. Cho tam giác $A B C$, các đường cao $A D, B E, C F$ cắt nhau tại trực tâm tam giác là $H$. Gọi $M$ là trung điểm $B C$ và $P$ là hình chiếu vuông góc của $H$ trên $A M$. Chứng minh rằng
(a) Các tứ giác $B F P M, C E P M$ nội tiếp.
(b) Tứ giác $B H P C$ nội tiếp.
(c) $B C$ là tiếp tuyến chung của đường tròn ngoại tiếp tam giác $A P B, A P C$.

Hướng dẫn - Gợi ý

(a) Ta chứng minh được
$$
A P \cdot A M=A H \cdot A D=A F \cdot A B=A E \cdot A C
$$
suy ra $B F P M, C E P M$ là các tứ giác nội tiếp.
(b) Ta có $\angle B H C=180^{\circ}-\angle B A C$ và
$$
\angle B P C=\angle B P M+\angle C P M=\angle B F M+\angle C E M=\angle A B C+\angle A C B=180^{\circ}-\angle B A C
$$

Suy ra $\angle B H C=\angle B P C$ nên $B H P C$ nội tiếp.
(c) Ta có
$$
\angle M P B=\angle A B M \Rightarrow \angle M B P=\angle M A B
$$
do đó $B M$ là tiếp tuyến của đường tròn ngoại tiếp tam giác $A B P$. Tương tự thì $M C$ cũng là tiếp tuyến của đường tròn ngoại tiếp tam giác $A P C$.

Bài 4. Cho tam giác $A B C$ nhọn nội tiếp đường tròn $(O)$. Các đường cao $B E, C F$ cắt nhau tại $H$. Gọi $M$ là trung điểm $A H$.
a) Chứng minh $M E D F$ nội tiếp.
b) $M E, M F$ là tiếp tuyến của đường tròn đường kính $B C$.
c) Gọi $K$ là giao điểm $A D$ và $E F ; T$ là giao điểm của $M B$ và đường tròn đường kính $B C$. Chứng minh rằng $T, K, C$ thẳng hàng và $K$ là trực tâm tam giác $M B C$.

Hướng dẫn - Gợi ý

(a) Gọi $I$ là trung điểm $B C$, thì $I$ là tâm đường tròn đường kính $B C$.
Ta có $\angle A E H=\angle A F H=90^{\circ}$ nên $A E H F$ nội tiếp và $M$ là tâm đường tròn.
Tam giác $M A E$ và $I C E$ cân nên $\angle M E A=\angle M A E, \angle I E C=\angle I C E$, suy ra $\angle M E A+\angle I E C=$ $\angle M A E+\angle I C E=90^{\circ}$, do đó $\angle M E I=90^{\circ}$, hay $M E$ là tiếp tuyến của đường tròn $(I)$ đường kính $B C$.
Tương tự thì $M F$ cũng là tiếp tuyến của đường tron2h $(I)$ đường kính $B C$.
(b) Ta có $\angle I E M=\angle I M F=\angle I D M=90^{\circ}$, do đó 5 điểm $I, M, D, E, F$ cùng thuộc đường tròn đường kính $I M$. Do đó tứ giác $M E D F$ nội tiếp.
(c) Gọi $L$ là giao điểm $I M$ và $E F$ thì ta có $I M \perp E F$ tại $L$. Khi đó ta có $M L \cdot M I=M E^2$ (1) Hơn nữa $\triangle M E T \backsim \triangle M B E$, suy ra $M T \cdot M B=M E^2$ Và $\triangle M K L \backsim \triangle M D I$, suy ra $M K \cdot M D=M L \cdot M I$ Từ (1), (2), (3) ta có $M T \cdot M B=M K \cdot M D$, từ đó ta có $B T K D$ nội tiếp, suy ra $\angle B T K=
180^{\circ}-\angle B D K=90^{\circ} \text {. }
$

Mặt khác $\angle B K C=90^{\circ}$, từ đó $T, K, C$ thẳng hàng.
Tam giác $M B C$ có hai đường cao $M D$ và $C T$ cắt nhau tại $K$ nên $K$ là trực tâm của tam giác.

Bài 5. Cho tam giác $A B C$ nhọn nội tiếp đường tròn $(O)$. Các đường cao $B E, C F$ cắt nhau tại $H$. Gọi $M$ là trung điểm $B C$.Đường tròn đường kính $A H$ cắt $(O)$ tại $P$ khác $A$. $A P$ cắt $B C$ tại $K$.
a) Chứng minh các tứ giác $K B F P, K C E P$ nội tiếp.
b) Chứng minh $K, E, F$ thẳng hàng.
c) Chứng minh $H$ là trực tam giác $A K M$.

Hướng dẫn - Gợi ý

(a) Ta có các tứ giác $A P F E, B F E C$ nội tiếp, suy ra $\angle K P F=\angle A E F, \angle A E F=\angle C B F$, từ đó $\angle K P F=\angle C B F$, kéo theo tứ giác $K B F P$ nội tiếp. Tương tự thì $\angle A P E=\angle A F E=\angle A C K$, do đó $K P E C$ nội tiếp.
(b) Tứ giác $\angle K P F B, K P E C$ nội tiếp, suy ra $\angle P K F=\angle P B F, \angle P K E=\angle A C P$ mà $\angle P B F=$ $\angle A C P$ nên $\angle P K F=\angle P K E$, từ đó ta có $K, F, E$ thẳng hàng.
(c) Ta có $A P F E$ nội tiếp và $A F H E$ nội tiếp nên $A, P, F, H, E$ cùng thuộc một đường tròn, suy ra $\angle A P H=\angle A F H=90^{\circ}$;
Vẽ đường kính $A D$, ta có $\angle A P D=90^{\circ}$; Do đó $P, H, D$ thẳng hàng.
Mặt khác, theo bài tập 1 thì $H, M, D$ thẳng hàng nên $P, H, M$ thẳng hàng và $M P \perp A K$. Tam giác $A K M$ có $A H, P M$ là hai đường cao cắt nhau tại $H$ nên $H$ là trực tâm tam giác, do đó $K H \perp A M$.

Bài 6. Cho tam giác $A B C$, các đường cao $B E, C F$. Các điểm $P \in B E, Q \in C F$ sao cho $\angle P A B=$ $\angle Q A C=90^{\circ}$. Chứng minh rằng đường thẳng qua $A$ vuông góc $P Q$ đi qua trung điểm $B C$.

Hướng dẫn - Gợi ý

Bài 7. Cho tam giác $A B C$ nội tiếp đường tròn tâm $O$ có trực tâm $H$. Đường trung trực $A H$ cắt $A B, A C$ tại $Q, P$. Chứng minh $O A$ là phân giác $\angle P O Q$.

Hướng dẫn - Gợi ý
  • $\angle B A H=\angle O A C, \angle B A O=\angle C A H$;
  • Ta có thể suy nghĩ hướng chứng minh $\triangle A O Q \backsim \triangle A H B$.

Theo nhận xét trên, thì ta cần chứng minh $A O \cdot A H=A B \cdot A Q$, thực vậy ta có hai tam giác $Q A H$ và $O A B$ cân tại $Q, O$ và $\angle Q A H=\angle O A B$ nên
$$
\triangle Q A H \backsim \triangle O A B \Rightarrow A O \cdot A H=A Q \cdot A B
$$

Từ đó ta có $\triangle A O P \backsim \triangle A B H$, kéo theo $\triangle A O Q=\angle A B H$
Chứng minh tương tự thì $\angle A O P=\angle A C H$
Từ (1) và (2) ta có $\angle A O Q=\angle A O P$.

Bài 8. Cho tam giác $A B C$ nội tiếp đường tròn $(O)$, các đường cao $B D, B E, C F$ cắt nhau tại $H$. $A D$ cắt $(O)$ tại $K . K F$ cắt $(O)$ tại $P$.
a) Chứng minh $F H \cdot F C=F P \cdot F K$.
b) $C P$ cắt $D E$ tại $L$. Chứng minh $H L P F$ nội tiếp.
c) Chứng minh $C P$ qua trung điểm của $E F$.

Hướng dẫn - Gợi ý

(a) Ta có tứ giác $A B D E$ nội tiếp nên $\angle C E D=\angle A B C$ mà $\angle A B C=\angle A P C$, suy ra $\angle A P C=$ $\angle C E D$, từ đó có tứ giác $A P L E$ nội tiếp. Khi đó
$$
C L \cdot A P=C E \cdot C A=C H \cdot C F
$$
suy ra $H L P F$ nội tiếp.
(b) $H L$ cắt $A C$ tại $T$. Ta cần chứng minh $H T / / F E$ và $L$ là trung điểm $H T$. Ý đầu tiên ta có thể làm như sau: tứ giác $H F P L, A F H E, A P K C$ nội tiếp nên
$$
\angle C H L=\angle C P F=\angle C A K=\angle C F E
$$
suy ra $H T / / F E$.
Tiếp theo
$$
\angle T H E=\angle F E H=\angle D E H
$$

$$
\angle H E T=90^{\circ} \Rightarrow \angle L E T=\angle L T E
$$
do đó
$$
L H=L E=L T
$$

Khi đó
$$
\frac{H L}{F M}=\frac{C L}{C M}=\frac{L T}{M E} \Rightarrow M F=M E
$$

Bài 9. Cho tam giác $A B C$ có các đường cao $B D, C E$ cắt nhau tại $H$. Gọi $K$ là hình chiếu vuông góc của $H$ trên $D E . M$ là trung điểm của $B C, L$ là giao điểm của $A M$ và $D E$. Chứng minh 4 điểm $B, C, L, K$ cùng thuộc một đường tròn.

Hướng dẫn - Gợi ý

Nhận xét: Đây là bài toán khó đối với các em THCS, tuy vậy dựa vào các bài toán đã giải ở trên ta có thể kết nối để cho ra lời giải bài toán này.
Ở đây việc kéo dài $K L$ cắt $B C$, có lẽ là ý nghĩa tự nhiên nhất để sử dụng tam giác đồng dạng, suy ra tứ giác nội tiếp.
Gọi giao điểm đó là $T$, thì ta cần chứng minh $T K \cdot T L=T B \cdot T C$, hơn nữa từ bài toán 1.4 ta có tính chất $T H \perp A M$, đây có lẽ là chìa khóa để cho ta lời giải bài toán này.

Do có sử dụng bài toán phụ ở trên nên lời giải bài toán có thể viết ngắn gọn hơn, đi thi thì học sinh nhớ chứng minh lại các ý toán đã sử dụng.

  • Gọi $T$ là giao điểm của $E F$ và $B C ; A H$ cắt $B C$ tại $D$.
  • Ta chứng minh được $T H \perp A M$ (bài 1.4), tại $Q$. Khi đó các tứ giác $H K T D, H Q M D$ nội tiếp nên
    $$
    \angle T K D=\angle T H D=\angle A M D
    $$
    suy ra $K L M D$ nội tiếp. Hơn nữa có các tứ giác $B F E C, E F D M$ nội tiếp nên
    $$
    T K \cdot T L=T D \cdot T M=T E \cdot T F=T B \cdot T C
    $$
    do đó $B K L C$ nội tiếp.

Bài 10. Cho tam giác $A B C$ nhọn, $M$ trên cạnh $B C$. Trên các cạnh $A B, A C$ lấy điểm $D, E$ sao cho $M D=M B, M E=M C$. Gọi $H$ là trực tâm tam giác $M D E$. Chứng minh rằng 4 diểm $A, D, H, E$ cùng thuộc một đường tròn.

Hướng dẫn - Gợi ý

Nhận xét: Bài này ta dễ nghĩ tới việc chứng minh $\angle D H E=180^{\circ}-\angle B A C=\angle P H Q$, trong đó $B Q, C P$ là các đường cao.
Do đó chỉ cần chứng minh tam giác $H P D$ và $P Q E$ đồng dạng, tới đây tính toán một chút vì hai tam giác này đều là các tam giác vuông.

Gọi $K, L$ lần lượt là trung điểm $B D, C E$. Ta có
$$
D P=B P-B D=B C \cos B-2 B M \cos B=\cos B(B C-2 B M)=\cos B(M C-M B)
$$
tương tự thì
$$
E Q=E C-C Q=\cos C(M C-M B)
$$

Suy ra
$$
\frac{D P}{E Q}=\frac{\cos B}{\cos C}=\frac{P B}{C Q}=\frac{H P}{H Q}
$$

Do đó
$$
\triangle H P D \backsim \triangle H Q E \Rightarrow \angle D H P=\angle E H Q, \angle D H E=\angle P H Q=180^{\circ}-\angle B A C
$$
do đó từ giác $A D H E$ nội tiếp.

Bài tập rèn luyện

Bài 1. (Chuyên Tiền Giang) Cho tam giác nhọn $A B C$ có $A B<A C$ và nội tiếp đường tròn tâm $O$. Đường tròn tâm $K$ đường kính $B C$ cắt các cạnh $A B, A C$ lần lượt tại $E, F$. Gọi $H$ là giao điểm của $B F$ và $C E$.
a) Chứng minh tam giác $A E F$ và tam giác $A C B$ dồng dạng.
b) Gọi $A^{\prime}$ là điểm đối xứng của $A$ qua $O$. Chứng minh $A A^{\prime}$ vuông góc với $E F$.
c) Từ $A$ dựng các tiếp tuyến $A M, A N$ dến đường tròn $(K)$ với $M, N$ là các tiếp điểm. Chứng minh ba điểm $M, H, N$ thẳng hàng.

Bài 2. (Chuyên Thái Nguyên) Cho tam giác nhọn $A B C$ nội tiếp đường tròn $(O), A B<A C$, các đường cao $B D, C E$ cắt nhau tại $H$ ( $D$ thuộc $A C, E$ thuộc $A B$ ). Gọi $M$ là trung điểm của $B C$, tia $M H$ cắt đường tròn $(O)$ tại $N$.
a) Chứng minh rằng năm điểm $A, D, E, H, N$ cùng nằm trên một đường tròn.
b) Lấy điểm $P$ trên đoạn $B C$ sao cho $\widehat{B H P}=\widehat{C H M}, Q$ là hình chiếu vuông góc của $A$ trên đường thẳng $H P$. Chứng minh rằng tứ giác $D E N Q$ là hình thang cân.
c) Chứng minh rằng đường tròn ngoại tiếp tam giác $M P Q$ tiếp xúc với đường tròn $(O)$.

Bài 3. (Lê Quý Đôn – Bình Định ) Cho tam giác $A B C(A B<A C)$ có các góc đều nhọn, các đường cao $A D, B E, C F$ cắt nhau tại $H$. Đường thẳng $E F$ cắt đường thẳng $B C$ và $A D$ lần lượt tại $K$ và I. Qua $F$ kẻ đường thẳng song song với $A C$ cắt $A K, A D$ lần lượt tại $M$ và $N$. Gọi $O$ là trung điểm $B C$. Chứng minh
a) $D A$ là phân giác của $\widehat{F D E}$.
b) F là trung điểm của $M N$.
c) $O D \cdot O K=O E^2$ và $B D \cdot D C=O D \cdot D K$.

Bài 4. (Chuyên TPHCM – 2018) Cho tam giác $A B C(A B<A C)$ vuông tại $A$ có đường cao $A H$. Gọi $E, F$ lần lượt là hình chiếu của $H$ lên $A B, A C$.
a) Chứng minh rằng $B E \sqrt{C H}+C F \sqrt{B H}=A H \sqrt{B C}$.
b) Gọi $D$ là điểm đối xứng của $B$ qua $H$ và gọi $O$ là trung điểm của $B C$. Đường thẳng đi qua $D$ và vuông góc với $B C$ cắt $A C$ tại $K$. Chứng minh rằng $B K$ vuông góc với $A O$.

Bài 5. (PTNK) Cho tam giác $A B C$ nhọn. Một đường tròn qua $B, C$ cắt các cạnh $A B, A C$ lần lượt tại $E$ và $F ; B F$ cắt $C E$ tại $D$. Lấy điểm $K$ sao cho tứ giác $D B K C$ là hình bình hành.
a) Chứng minh rằng $\triangle K B C$ dồng dạng với $\triangle D F E, \triangle A K C$ dồng dạng với $\triangle A D E$.
b) Hạ $D M$ vuông góc với $A B, D N$ vuông góc với $A C$. Chứng minh rằng $M N$ vuông góc với $A K$.
c) Gọi $I$ là trung điểm $A D, J$ là trung điểm $M N$. Chứng minh đường thẳng $I J$ đi qua trung điểm của cạnh $B C$.
d) Đường thẳng $I J$ cắt đường tròn ngoại tiếp tam giác $I M N$ tại $T$ (khác $I$ ). Chứng minh rằng $A D$ tiếp xúc với đường tròn ngoại tiếp tam giác $D T J$.

Đường thẳng Simson của tam giác

Bài toán 1. Cho tam giác $ABC$ nội tiếp đường tròn $(O)$ và $P$ là điểm thuộc cung $AC$ không chứa $B$. Gọi $D, E, F$ lần lượt là hình chiếu của $P$ trên các đường thẳng $BC, AC, AB$.

a) Chứng minh các tứ giác $PCDE, PDBF$ nội tiếp.

b) Chứng minh $D, P, E$ thẳng hàng.

c) Chứng minh tam giác $PDE$ và $PBA$ đồng dạng; tam giác $PFE$ và $PBC$ đồng dạng.

Lời giải.

a) Tứ giác $PCDE$ có $\angle PDC = \angle PEC = 90^\circ $ nên là tứ giác nội tiếp.

Tứ giác $PDFB $ có $\angle PDB + \angle PFB = 90^\circ + 90^\circ = 180^\circ$ nên là tứ giác nội tiếp.

b) Ta có $\angle PFE = \angle PAE$ vì $PFAE$ nội tiếp

Mà $\angle PAE = \angle PBC = \angle PFD$;

Do đó $\angle PFE = \angle PFD$, suy ra $F, E, D$ thẳng hàng.

c) Xét tam giác $PDE$ và $PBA$ có $\angle PDE = \angle PCA = \angle PBA, \angle PED = 180^\circ – \angle PCB = \angle PAB$, do đó $\triangle PDE \backsim \triangle PBA$.

Chú ý: Cho tam giác $ABC$ và $P$ là một điểm bất kì thuộc đường tròn ngoại tiếp tam giác, khi đó hình chiếu của $P$ trên các đường thẳng $BC, AC, AB$ cùng thuộc một đường thẳng. Đường thẳng này được gọi là đường thẳng Simson của điểm $P$ đối với tam giác $ABC$.

Sau đây ta xem một số bài toán liên quan đến đường thẳng simson

Bài 1. Cho tam giác $A B C$ nhọn nội tiếp đường tròn $(O), P$ là điểm thay đổi trên cung $B C$ không chứa $A$. Gọi $D, E$ là hình chiếu vuông góc của $A$ trên $P B, P C$.
a) Tìm vị trí của $P$ để $A D \cdot P B+A E \cdot P C$ lớn nhất.
b) Chứng minh rằng $D E$ đi qua một điểm cố định. Tìm vị trí của $P$ để $D E$ lớn nhất.

Hướng dẫn

a) Ta có $AD \cdot BP +AE \cdot PC = 2S_{ABP} + 2S_{ACP} = 2S_{ABPC} = 2 (S_{ABC}+S_{PBC})$

Do đó $AD \cdot BP + AE \cdot PC $ lớn nhất khi $S_{PBC}$ lớn nhất, $P$ là điểm chính giữa cung $BC$.

b) Gọi $H$ là hình chiếu của $A$ trên $BC$, khi đó $D, E, H$ thẳng hàng, hay $DE$ qua $H$ cố định.

Bài 2. Cho tam giác $A B C$, nội tiếp đường tròn $(O), P$ là điểm thuộc cung $A C$, gọi $D, E$ là hình chiếu vuông góc của $P$ trên $B C, A C$.
a) $D E$ cắt $A B$ tại $F$. Chứng minh $P F \perp A B$.
b) Gọi $M, N$ lần lượt là trung điểm $A B, D E$. Tính $\angle P N M$.

Hướng dẫn

a) Tự giải

b) Tam giác $PDE$ và $PBA$ đồng dạng, $M, N$ lần lượt là trung điểm $AB, DE$ nên $PMB$ và $PNE$ đồng dạng, suy ra $\angle PNE = \angle PMB$, từ đó $PNFM$ nội tiếp, suy ra $\agle PNM = 90^\circ$.

Bài 3. Cho tam giác $A B C$ các đường cao $A D, B E, C F$. Gọi $M, N, P, Q$ lần lượt là hình chiếu vuông góc của $D$ trên $A B, A C, B E, C F$. Chứng minh $M, N, P, Q$ thẳng hàng.

Hướng dẫn

Tứ giác $BFHD$ nội tiếp, nên hình chiếu của $D$ trên $BF, BH, FH$ thẳng hàng, hay $M, P, Q$ thẳng hàng. Tương tự cho tứ giác $CDHE$ thì $N, P, Q$ thẳng hàng.

Bài 4. Cho tam giác $A B C$ nội tiếp đường tròn $(O), P Q$ là đường kính của $(O)$.

a) Chứng minh rằng đường thẳng simson của $P, Q$ ứng với tam giác $A B C$ thì vuông góc nhau tại $I$.

b) Chứng minh $I$ thuộc đường tròn Euler của tam giác $ABC$.

Hướng dẫn

a) Xét hình như hình trên, ta có $\angle EDC = \angle EPC$ và $\angle LKB = \angle LQC$

Suy ra $\angle EDC + \angle LKB = \angle EPC + \angle LQC = 90^\circ – \angle ECP + 90^\circ – \angle QCL = 180^\circ – \angle QCP = 90^\circ$, do đó tam giác $DIK$ vuông tại $I$ hay $DE \bot LK$ tại $I$.

b) Gọi $M, N, P$ lần lượt là trung điểm $BC, AC, AB$. Do $O$ là trung điểm $PQ$ nên $M$ cũng là trung điểm $DK$, $N$ là trung điểm $LE$.

Khi đó $IN =IL = IE, IM = ID = IK$, suy ra $\angle LIN = \angle ILN = \angle CQK, \angle DIM = \angle MDK = \angle EPC$.

Do đó $\angle MIN = 90^\circ + \angle LIN + \angle DIM = 90^\circ + \angle CQK + \angle EPC = 180^\circ – \angle ACB = 180^\circ – \angle MPN$, do đó $IMPN$ nội tiếp, hay $I$ thuộc đường tròn Euler của tam giác $ABC$.

Bài 5. (IMO 2007) Xét 5 diểm $A, B, C, D, E$, sao cho $A B C D$ là hình bình hành và $B, C, D, E$ cùng thuộc một đuoòng tròn. Gọi $d$ là đuoòng thẳng qua $\mathrm{A}$, giả sủ $d$ cắt đoạn $B C$ tại $F$ và $B C$ tại $G$. Giả sủ $E F=E G=E C$, chúng minh rằng $\mathrm{d}$ là phân giác của $\angle D A B$.

Hướng dẫn

Gọi $I, H$ là trung điểm của $C G, C F$. Ta có $E I \perp C G, E H \perp C F$. Ta có $O, H, I$ thẳng hàng do $O H, O I$ cùng song song với $d$.

Dễ dàng chứng minh được $E O \perp B D$. Suy ra tam giác $E B D$ cân. Từ đó suy ra $C E$ là phân giác góc $\angle B C G$ và $d$ là phân giác $\angle D A B$.

Bài 6. Cho tứ giác $A B C D$ nội tiếp. Gọi $d_a$ là đường thẳng simson của tam giác $B C D$ ứng với điểm $A$; các đường thẳng $d_b, d_c, d_d$ xác định tương tự. Chứng minh rằng $d_a, d_b, d_c, d_d$ đồng quy.

Hướng dẫn
  • Gọi $H_a, H_b$ là trự tâm tam giác $BCD, ACD$.
  • Chứng minh $d_a$ qua trung điểm $AH_a$;
  • Chứng minh $AH_aH_bB$ là hình bình hành.

Một số bài đường tròn và tiếp tuyến

Bài 1. Cho đường tròn tâm $O$ đường kính $A B$. $C$ là một điểm thuộc đường tròn. $d_1$ và $d_2$ lần lượt là tiếp tuyến tại $A$ và $B$ của $(O)$. Tiếp tuyến tại $C$ cắt $d_1, d_2$ lần lượt tại $D$ và $E$. $B C$ cắt $d_1$ tại $F$.
a) Chứng minh $d_1 | d_2$ và $D$ là trung điểm của $A F$.
b) Vẽ đường cao $C H$. Chứng minh rằng $A E, B D$ và $C H$ dồng quy tại trung điểm của $C H$.
c) Chứng minh $O F \perp A E$.

Lời giải.

a) $d_1$ là tiếp tuyến tại $A$ nên $O A \perp d_1, d_2$ là tiếp tuyến tại $B$ nên $d_2 \perp O B$, mà $O, A, B$ thẳng hàng, suy ra $d_1 / / d_2$.
Ta có $\angle A C B=90^{\circ}$, suy ra $\angle D C F+$ $\angle D C A=\angle D F C+\angle D A C=90^{\circ}$. (1)
Hơn nữa $D A=D C$ (t/c tiếp tuyến), tam giác $D A C$ cân tại $D$, suy ra $\angle D C A=$ $\angle D A C$. (2)
Từ (1) và (2) ta có $\angle D C F=\angle D F C$, tam giác $D C F$ cân tại $D$.
Vậy $D F=D C=D A$, hay $D$ là trung điểm của $A F$.
b) Gọi $I$ là giao điểm của $B D$ và $A E$. Ta có $A D / / B E$ nên $\frac{B I}{I D}=\frac{E B}{A D}(3)$.
Mặt khác do $A D=D C$ và $E B=E C$, suy ra $\frac{E B}{A D}=\frac{E C}{D C}$ (4).

Từ (3) và (4) ta có $\frac{B I}{I D}=\frac{E C}{D C}$, suy ra $I C / / A D$ (Thalet đảo).

Mà $A D \perp A B$ nên $C I \perp A B$, vậy $C, I, H$ thẳng hàng.

Do đó $A E, B E, C H$ đồng quy tại $I$.
Ta có $\frac{C I}{A D}=\frac{E I}{E A}, \frac{I H}{A D}=\frac{B I}{B D}$ và $\frac{E I}{E A}=$ $\frac{B I}{B D}$, nên $\frac{C I}{A D}=\frac{I H}{A D}$, suy ra $I C=I H$ hay
$I$ là trung điểm của $C H$.
c) Ta có $E B \cdot A D=E C \cdot C D=O C^2=R^2$, mà $A F=2 A D$ nên $E B \cdot A F=2 R^2$.

Suy ra $E B \cdot A F=A O \cdot A B$, suy ra $\frac{E B}{A B}=\frac{O A}{A F}$, do đó $\tan E A B=\tan A F O$, suy ra $\angle E A B=$ $\angle A F O$.
Mà $\angle E A B+\angle E A F=90^{\circ}$ nên $\angle E A B+$ $\angle A F O=90^{\circ}$. Do đó $O F \perp A E$.

Bài 2. Cho đường tròn tâm $O$ bán kính $R$. $A$ là một điểm nằm ngoài đường tròn, từ $A$ dựng các tiếp tuyến $A B, A C$ dến $(O)$ với $B, C$ là các tiếp điểm. Một cát tuyết qua $A$ cắt $(O)$ tại $D$ và $E$ trong đó $D$ nằm giữa $A$ và $E$.Gọi $H$ là giao điểm của $O A$ và $B C$.
a) Chứng minh $O H \cdot O A=R^2$.
b) Gọi $M$ là trung điểm của $D E$. Chứng minh 4 điểm $O, M, B, C$ cùng thuộc đường tròn.
c) Tiếp tuyến tại $D$ và $E$ của $(O)$ cắt nhau tại điểm $P$. Chứng minh $P, B, C$ thẳng hàng.

Lời giải.

a) Ta có $A B, A C$ là tiếp tuyến nên $A B=A C$, và $O B=O C=R$, suy ra $O A$ là trung trực của $B C$, suy ra $O A \perp B C$ tại $H$.
Tam giác $O A B$ có $\angle O B A=90^{\circ}$ (t/c tiệp tuyến) và $B H \perp O A$ nên $O H \cdot O A=O B^2=$ $R^2$.
b) $M$ là trung điểm $D E$, suy ra $O M \perp D E$.
Ta có $\angle O B A=\angle O M A=\angle O C A=90^{\circ}$, suy ra 5 diểm $O, M, B, A, C$ cùng thuộc đường tròn đường kính $O A$.
c) Ta chứng minh được $O P \perp D E$, suy ra $O, M, P$ thẳng hàng và $O M . O P=O D^2=$ $R^2$.
Suy ra $O M \cdot O P=O H \cdot O A$, suy ra $\frac{O M}{O H}=$ $\frac{O P}{O A}$.
Xét tam giác $O M A$ và tam giác $O H P$ có:
$\angle A O P$ chung $\frac{O M}{O H}=\frac{O P}{O A}$ $\angle O H P=\angle O M A=90^{\circ}$.
Ta có $B C, P H$ vuông góc với $O A$ tại $H$ nên $P, B, C$ thẳng hàng.

Bài 3. Cho tam giác $A B C$ vuông tại $A(A B<A C)$. Vẽ đường tròn tâm $O$ đường kính $A C$ cắt cạnh $B C$ tại $D$. Gọi $H$ và $K$ lần lượt là trung điểm của hai cạnh $A D$ và $C D$. Tia $O H$ cắt cạnh $A B$ tại $E$. Tia $O K$ cắt đường thẳng $E D$ tại $N$ và cắt đường tròn tâm $O$ tại $I$.
(a) Chứng minh $D E$ là tiếp tuyến của $(O)$.
(b) Chứng minh $O H D K$ là hình chữ nhật.
(c) Chứng minh tia $D I$ là tia phân giác của $\angle N D C$.
(d) Gọi $S$ là giao điểm của $O B$ với $A D$. Từ $S$ vẽ đường thẳng vuông góc với $A O$ và cắt tia $O H$ tại $Q$. Chứng minh 3 điểm $A, Q, N$ thẳng hàng.

Lời giải.

Hình 1

a) $OH$ là trung trực của $AD$, suy ra $EA = ED$. Từ đó $\triangle EDO = \triangle EAO (ccc)$, suy ra $\angle EDO = \angle EAO = 90^\circ$. Do đó $ED$ là tiếp tuyến của $(O)$.

b) Do $K$ là trung điểm $CD$ nên $OK \bot CD$, tứ giác $OHDK$ có $\angle D = \angle H = \angle K = 90^\circ$ nên là hình chữ nhật.

c) Ta có tam giác $ODI$ cân tại $O$ nên $\angle ODI = \angle OID$ (1)
Mà $\angle ODI = \angle ODK + \angle KDI, \angle OID = \angle OND + \angle NDI$ (2)
Và $\angle OND = \angle ODK$ (vì cùng phụ $\angle DON$) (3)
Từ (1), (2), (3) ta có $\angle KDI = \angle NDI$

d) Gọi $L$ là giao điểm $AQ$ và $OS$.
Trong tam giác $ASO$ có $AQ, SQ$ là các đường cao, nên $Q$ là trực tâm, suy ra $AQ \bot OS$ tại $L$. (4)
Ta có $OL \cdot OB = OA^2$
và $OK \cdot ON = OD^2 = OA^2$
Suy ra $\angle OK \cdot ON = OL \cdot OB$
Suy ra $\triangle OLN \backsim \triangle OKB$, suy ra $\angle OLN = \angle OKB = 90^\circ$ (5)
Từ (4), (5) ta có $A, L, N$ thẳng hàng, hay $A, Q, N$ thẳng hàng.

Bài 4. Cho đường tròn $(O ; R)$ và một điểm $S$ nằm ngoài đường tròn $(O)$. Vẽ hai tiếp tuyến $S B, S C$ đến $(O)$ với $B, C$ là hai tiếp điểm. Gọi $H$ là giao điểm của $S O$ với $B C$.
(a) Vẽ đường kính $B A$ của $(O)$. Chứng minh $A C || S O$ và $H B \cdot H C=H O \cdot H S$.
(b) Vẽ đường thẳng $d$ vuông góc vớ $A B$ tại $O$, đường thẳng $d$ cắt đường thẳng $A C$ tại $E$. Chứng minh $S E=R$.
(c) Vẽ $C K$ vuông góc với $A B$ tại $K$. Gọi $I$ là trung điểm của cạnh $C K$. Chứng minh 3 điểm $S, I, A$ thẳng hàng.

Lời giải.

a) Do $AB$ là đường kính của $(O)$ nên $\angle ACB = 90^\circ$. (1)

Ta có $SB = SC$ và $SO$ phân giác $\angle BSC$ nên $SO$ là trung trực của $BC$, do đó $OS \bot BC$ tại $H$.

Từ đó ta có $AC ||OS$ vì cùng vuông góc $BC$.

b) $\triangle AOE = \triangle OBS (gcg)$, suy ra $OE = BS$.

Tứ giác $OESB$ có $OE||BS$ (Cùng vuông góc $AB$), và $OE = BS$ nên $OESB$ là hình bình hành, hơn nữa có $\angle OBS= 90^\circ$ nên là hình chữ nhật, do đó $SE = OB = R$.

c) Ta có $OASE$ là hình bình hành, suy ra $AS$ cắt $OE$ tại trung điểm $T$ của mỗi đoạn.
$CK ||OE$
Gọi $I’$ là giao điểm của $AS$ và $CK$
Ta có $\dfrac{I’K}{OT} = \dfrac{AI’}{AT} = \dfrac{CI’}{ET}$
Mà $OT = ET$ nên $KI’ = CI’$, hay $I’ \equiv I$
Vậy $A, I, S$ thẳng hàng

Bài 5. Cho đường tròn $(O ; R)$ và điểm $M$ ở ngoài đường tròn $(O)$. Kẻ tiếp tuyến $M A, M B$ đến $(O)$ với $A, B$ là hai tiếp điểm. Đường thẳng $A B$ cắt $(O)$ tại $K$.
(a) Kẻ đường kính $A N$ của $(O), B H \perp A N$ tại $H$. Chứng $\operatorname{minh} M B \cdot B N=B H \cdot M O$.
(b) Đường thẳng $M O$ cắt đường tròn $(O)$ tại $C$ và $D(C$ nằm giữa $O$ và $M)$. Chứng minh $O K \cdot M K=C K \cdot D K$.
(c) $E$ đối xứng với $C$ qua $K$. Chứng minh $E$ là trực tâm của tam giác $A B D$.
(d) Chứng minh $\sin \angle M^{\circ} A B=\frac{C K}{A K}+\frac{C K}{A M}$

Lời giải.

a) Chứng minh tam giác $OMB$ và $NBH$ đồng dạng.
b) $OK \cdot MK = AK^2 = KC \cdot KD$
c) $ACBE$ là hình thoi, suy ra $BE||AC$, mà $AC \bot AD$ suy ra $BE \bot AD$
$DE \bot AB$
Do đó $E$ là trực tâm tam giác $ABD$.

d) $\angle CAK = \angle CAM$ (chứng minh ở bài trên)
Do đó $\dfrac{CK}{CM} = \dfrac{AK}{AM}$, suy ra $\dfrac{CK}{AK} = \dfrac{CM}{AM}$
Từ đó $VP = \dfrac{CK}{AK} + \dfrac{CK}{AM} = \dfrac{CM}{AM} + \dfrac{CK}{AM} = \dfrac{KM}{AM} = \sin MAB$

Bài 6. Cho hình vuông $A B C D$ cạnh $a, E$ là cung thuộc cung nhỏ $B D$ của đường tròn tâm tâm $A$ bán kính $a$. Tiếp tuyến tại $E$ cắt $C D$ tại $F$ và $B C$ tại $G$.
(a) Chứng minh chu vi tam giác $C F G$ bằng $2 a$.
(b) $A F, A G$ cắt $B D$ tại $I$ và $H$. Chứng minh $H E=$ $H B, I E=I D$

và $H I^2=D I^2+B H^2$
(c) Chứng minh $F H, G I$ và $A E$ đồng quy.

Lời giải.

a) $CD, CB, FG$ là tiếp tuyến của $(A;a)$
Suy ra $FE = FD, GE = GB$
$P_{CFG} = CF + FG + CG = CF + EF +EG+CG = CF+DF +GB+CG = CD+ CB = 2a$

b) $AF$ là trung trực $DE$, và $AG$ là trung trực $BE$
Suy ra $IE = ID, HB = HE$
$\triangle IEF = \triangle IDF \Rightarrow \angle IEF =\angle IDF = 45^\circ$
Tương tự cũng có $\angle HEG = 45^\circ$
Suy ra $\angle IEH = 90^\circ$
Áp dụng pitago cho tam giác $EIH$ ta có $IH^2 = IE^2 + HE^2 = ID^2 + HB^2$

c) Ta có $AF$ là phân giác $\angle DAE$, $AG$ là phân giác của $\angle BAE$
Suy ra $\angle FAG = \dfrac{1}{2} \angle BAD = 45^\circ$.
$\triangle AIH \backsim \triangle DIF (gg)$, suy ra $IA \cdot IF = ID \cdot IH$
Suy ra $\triangle IFH \backsim \triangle IDA \Rightarrow \angle IFH = \angle IDA = 45^\circ$
Suy $\angle AHF = 90^\circ$ hay $FH \bot AG$.
Chứng minh tương tự $GI \bot AF$.
Tam giác $FG$ có $AE, FH, GI$ là các đường cao nên đồng quy.

Bài 7. (Cuối khóa 1 – Star Education 2018) Cho đường tròn $(O ; R)$ và điểm $A$ nằm ngoài đường tròn. Từ $A$ vẽ các tiếp tuyến $A B, A C$ dến $(O)$ ( $B, C$ là các tiếp điểm). $O A$ cắt $B C$ tại $H$.
a) Chứng minh $O H \cdot O A=R^2$ và 4 điểm $O, A, B, C$ cùng thuộc một đường tròn.
b) Đường tròn tâm $I$ đường kính $A B$ cắt $(O)$ tại điểm $D$ khác $B$. Chứng minh $I D$ là tiếp tuyến của $(O)$.
c) Tiếp tuyến tại $H$ và tại $A$ của $(I)$ cắt nhau tại $P$. Chứng minh $B, D, P$ thẳng hàng.

d) Tiếp tuyến tại $H$ của $(I)$ cắt $O B$ tại $M$; gọi $N$ là trung điểm $P M$, đường thẳng qua $P$ song song $B N$ cắt $A B$ tại $K$. Chứng minh $H K, A M$ và $B D$ đồng quy.

Lời giải.

a)

Xét $\triangle A B O$ vuông tại $B$ có:

$B H$ là đường cao $\Rightarrow O H \cdot O A=O B^2=R^2$ (Hệ thức lượng)

Ta có: $\triangle A B O$ vuông tại $B \Rightarrow A, B, O$ thuộc đường tròn đường kính $A O$. (1)

Lại có $\triangle A C O$ vuông tại $C \Rightarrow A, C, O$ thuộc đường tròn đường kính $A O$. (2)

Từ (1) và (2) suy ra $A, B, O, C$ thuộc đường tròn đường kính $A O$.

b)

Ta có: $\triangle A B D$ nội tiếp đường tròn đường kính $A B \Rightarrow \triangle A B D$ vuông tại $D$

Mà $I$ là trung điểm cạnh huyền $A B \Rightarrow I B=I D$
Ta có: $I B=I D, O B=O D$ nên $I O$ là trung trực của $B D$ $\Rightarrow \angle I B O=\angle I D O=90^{\circ}$ nên $I D$ là tiếp tuyến của $(O)$.

c) Tiếp tuyến tại $H$ và tại $A$ của $(I)$ cắt nhau tại $P$. Chứng minh $B, D, P$ thẳng hàng.

Gọi $E=I P \cap A H$ và $F=I O \cap B D$.
Sử dụng tính chất hai tiếp tuyến cắt nhau và hệ thức lượng, ta chứng minh được
$$
I E \cdot I P=I A^2=I D^2=I F \cdot I O \Rightarrow \frac{I F}{I P}=\frac{I E}{I O}
$$

Từ đó, chứng minh được $\triangle I F P \backsim \triangle I E O$ (c.g.c)
$$
\Rightarrow \angle I E O=\angle I F P=90^{\circ} \text {. }
$$

Ta có: $B D$ đi qua $F$ và vuông góc $I O, F P$ đi qua $F$ và vuông góc $I O$ nên hai đường thẳng này trùng nhau. $\Rightarrow B, D, P$ thẳng hàng.

d)

Chứng minh $I H$ là đường trung bình của $\triangle A B C \Rightarrow I H || A C$. Mà $I H \perp P M$ và $A C \perp O C$.

Suy ra: $H M || O C$. Lại có $H$ là trung điểm $B C$ nên $M$ là trung điểm $O B$.

Gọi $Q$ là giao điểm của $P K$ và $B O$.
Ta có: $B N || P Q$ và $N$ là trung điểm của $P M$ nên suy ra $B$ là trung điểm của $Q M$.

Gọi $J=B P \cap A M$.
Ta có :
$ B Q ||A P \Rightarrow \frac{B K}{K A}=\frac{B Q}{P A}=\frac{B M}{P A} . $
$B M || A P \Rightarrow \frac{B M}{P A}=\frac{B J}{J P}$
Suy ra: $\frac{B K}{K A}=\frac{B J}{J P}$ nên $K J || A P$. Chứng minh tương tự $J H ||A P$. Từ đó ta có $K, J, H$ thẳng hàng.

Vậy $H K, B P, A M$ dồng quy tại $J$.

Bài tập luyện tập.

Bài 6. Cho tam giác $A B C$ nhọn. Các đường cao $A D, B E$ và $C F$ cắt nhau tại $H$. Gọi $M, N$ lần lượt là trung điểm của $B C$ và $A H$.
(a) Chứng minh $N E, N F$ là tiếp tuyến của đường tròn ngoại tiếp tam giác $B C E$.
(b) Chứng minh 5 điểm $D, E, F, M, N$ cùng thuộc một đường tròn.
(c) Gọi $G$ là giao điểm của $A D$ và $E F$. Chứng minh $N G \cdot N D=N A^2$.

Bài 7. Cho nửa đường tròn tâm $O$ đường kính $A B=2 R$. Trên tiếp tuyến tại $A$ của $(O)$ lấy điểm $C$ sao cho $A C=A B$. Từ $C$ vẽ tiếp tuyến $C D$ dến $(O)$ cắt tiếp tuyến tại $B$ ở điểm E.
(a) Tính $B E$.
(b) Đường cao $D F$ của tam giác $A B D$ cắt $B C$ tại $G$. Chứng minh rằng $A, G, E$ thẳng hàng.
(c) Gọi $H$ là giao điểm của $O C$ và $A D$. Tính $\angle D H B$.
(d) Gọi $I$ là giao điểm của $B C$ và $(O)$. Tứ giác $I D B H$ là hình gì? Tại sao?

Bài 8. Cho tam giác $A B C$ nhọn nội tiếp đường tròn $(O) . M$ là trung điểm $B C$. Từ $A$ dựng các tiếp tuyến đến đường tròn $(O ; O M)$ cắt $B C$ tại $D$ và $E$ sao cho $D$ và $C$ khác phía đối với $M ; E, B$ khác phía đối với $M$. Chứng minh rằng các tam giác $A D C$ và $A B E$ cân.

Bài 9. Cho tam giác $A B C$ vuông tại $A, A B=a, B C=2 a$. Đường cao $A H$. Từ $B, C$ vẽ các tiếp tuyến $B D, C E$ dến đường tròn tâm $A$ bán kính $A H$.
(a) Tính $A H$ và số đo $\angle A B C$.
(b) Chứng minh $D, A, E$ thẳng hàng.
(c) Chứng minh $E D$ là tiếp tuyến của đường tròn đường kính $B C$.
(d) Chứng minh $D C, B E$ và $A H$ dồng quy.

Bài 10. Cho hình vuông $A B C D$ cạnh $2 a$, tâm $O$. Đường tròn tâm $O$ bán kính $a$ tiếp xúc với $A B$ và $B C$ tại $E$ và $F$. Gọi $P$ là một điểm trên cung nhỏ $E F$. Tiếp tuyến tại $P$ cắt $A B, B C$ tại $M$ và $N$. Đặt $M B=c, B N=y$.
(a) Chứng minh rằng $x+y+\sqrt{x^2+y^2}=2 a$.
(b) Chứng minh rằng $A M \cdot C N=2 a^2$.
(c) Gọi $K$ là trung điểm của $A D$. Chứng minh rằng $M K |$ $D N$.

Sử dụng phương pháp điểm trùng để chứng minh hình học

Trong việc giải các bài toán hình học, có một kĩ thuật khá là đặc biệt và cũng thường được sử dụng đó là sử dụng điểm trùng, kĩ thuật này dựa trên sự xác định duy nhất của hình để thực hiện.

Tình huống thường gặp nhất, ta cần chứng minh tính chất hay sự tồn tại của một số đối tượng hình học, chẳng hạn như giao điểm của một số đường thẳng. Khi đó, gọi hai hay một số giao điểm (dĩ nhiên tồn tại) của một số cặp hay một số đối tượng. Sau đó, ta sẽ chứng minh các giao điểm (đối tượng) mà ta vừa dựng là trùng nhau. Đôi khi để thực hiện điều này, ta cũng cần gọi thêm một số đối tượng khác cùng đi qua điểm đang xét rồi xét sự đồng quy của chúng với các đối tượng gọi thêm nhằm có thêm tính chất của các điểm mà ta cần chứng minh trùng nhau.

Ta chú ý một số tính chất sau:

Định lý 1. Về giao điêm của các đối tượng hình học:

  1. Hai đường thẳng có nhiều nhất 1 giao điêm.
  2. Hai đường tròn có nhiều nhất 2 giao điểm.
  3. Một đường thẳng và một đường tròn có nhiều nhất 2 giao điểm.
  4. Một tia có gốc nằm trong đường tròn và đường tròn đó có nhiều nhât 1 giao điềm.

Sau đây ta xét một số ví dụ trong chương trình toán hình học lớp 9.

Ví dụ 1. Cho đường tròn tâm $O$ đường kính $AB$, $C$ thuộc đường tròn. Tiếp tuyến tại $C$ cắt tiếp tuyến tại $A, B$ của $(O)$ tại $D, E$. Gọi $H$ là hình chiếu của $C$ trên $AB$.

a. $DB$ cắt $CH$ tại $N$. Chứng minh $A, N, E$ thẳng hàng.

b.Đường thẳng qua $A$ song song $HE$ và đường thẳng qua $B$ song song với $HD$ cắt nhau tại $M$. Chứng minh $D, M, E$ thẳng hàng.


a. $BC$ cắt $AD$ tại $F$, ta chứng minh được $D$ là trung điểm của $AF$.

Khi đó $\dfrac{CN}{DF} = \dfrac{PN}{PD} = \dfrac{HN}{AD}$.

Mà $AD = DF$, suy ra $CN = HN$ hay $N$ là trung điểm của $CH$.

Gọi $N’$ là giao điểm của $AE$ và $CH$, chứng minh tương tự ta cũng có $N’$ là trung điểm của $CH$. Do đó $N \equiv N’$ hay $A, N, E$ thẳng hàng.

b. Phân tích: vẽ hình chính xác và trực giác ta dự đoán được $M$ là trung điểm của $DE$, hơn nữa điểm $M$ là được xác định duy nhất do là giao điểm của 2 đường, do đó ta có thể gọi $M’$ là trung điểm và chứng minh $M’ \equiv M$ bằng cách chứng minh $AM’||HD$ và $BM’||HC$. Thực ra do vai trò như nhau nên chỉ cần chứng minh $AM’||HD$ là đủ.

Ta có $\dfrac{HA}{HB} = \dfrac{CD}{CE} = \dfrac{AD}{BE}$. Suy ra $\triangle AHD \backsim \triangle BHE$. Suy ra $\angle AHD = \angle BHE$

Suy ra $\angle KHA = \angle BHE = \angle AHD$. Từ đó ta có tam giác $HDK$ cân tại $H$ và $A$ là trung điểm $AD$.

Tam giác $DHE$ có $M’A$ là đường trung bình nên $AM’||EK$ hay $AM’||HE$.

Chứng minh tương tự ta có $BM’||HD$.

Vậy $M’ \equiv M$. Hay $D, M, E$ thẳng hàng.

Ví dụ 2. (LHP 2019) Cho tam giác đều $A B C$. Gọi $M, N$ là hai điểm nằm trên cạnh $B C$ sao cho $\angle M A N=30^{\circ}(M$ nằm giữa $B$ và $N)$. Gọi $K$ là giao điểm của hai đường tròn $(A B N)$ và $(A C M)(K$ khác $A)$. Chứng minh rằng hai điểm $K$ và $C$ đối xứng với nhau qua $A N$.

Lời giải

Việc chứng minh trực tiếp $K, C$ đối xứng qu $AN$ nhìn có vẻ dễ nhưng khi tìm cách chứng minh thì liên kết lại hơi khó, cảm giác như bị thiếu thiếu gì đó, ta phải vẽ thêm yếu tố phụ mới có thể làm được. Do đó ta nghĩ tới kĩ thuật điểm trùng, tức là dựng ra một điểm $K’$ đối xứng với $C$ qua $AN$ và chứng minh $K’$ là giao điểm của hai đường tròn.

Gọi $K$ là điểm đối xứng của $C$ qua $A N$. Có
$$
\angle A K^{\prime} N=\angle A C N=\angle A B N
$$
nên tứ giác $A B K^{\prime} N$ nội tiếp. Suy ra $K^{\prime} \in(A B N)$. Có
$$
\angle M A K^{\prime}+\angle N A C=\angle M A K^{\prime}+\angle K^{\prime} A N=30^{\circ}
$$
$$
\angle B A M+\angle N A C=30^{\circ}
$$
suy ra $\angle M A K^{\prime}=\angle B A M$.
Suy ra $\triangle A B M=\triangle A K^{\prime} M(c-g-c)$ nên $\angle A K^{\prime} M=\angle A B C=\angle A C B$ ta thu được $K^{\prime} \in(A M C)$. Vậy $K \equiv K^{\prime}$ ta có điều phải chứng minh.

Ví dụ 3. Cho tam giác $ABC$ nhọn nội tiếp đường tròn $(O)$, có $H$ là trực tâm tam giác $ABC$ và $AD$ là đường kính của $(O)$. Trên các cạnh $AB, AC$ lấy $E, F$ sao cho $AE = AF$ và $E, H, F$ thẳng hàng. Đường tròn ngoại tiếp tam giác $AEF$ cắt phân giác góc $\angle BAC$ tại $P$. Chứng minh $H, P, D$ thẳng hàng.

Lời giải

Gọi $P’$ là giao điểm phân giác góc $\angle BAC$ và $HD$. Ta chứng minh $P’ \equiv P$, hay cần chứng minh $AEPF$ nội tiếp.

Ta có tính chất quen thuộc $\angle HAB = \angle DAC$, nên $AP’$ cũng là phân giác $\angle HAD$.

Ta có $\angle AEF = \angle ABH + \angle EHB$, $\angle AFE = \angle ACH + \angle FHC$.

Mà $\angle ABH = \angle ACH$ và $\angle AEF = \angle AFE$ nên $\angle EHB = \angle FHC = \angle EHL$.

Do đó $HE$ là phân giác $\angle LHB$, suy ra $\dfrac{LE}{EB} = \dfrac{HL}{HB}$. (1)

Tam giác $AHL $ và tam giác $ADC$ đồng dạng, suy ra $\dfrac{HL}{CD} = \dfrac{AH}{AD}$.

Mà $CD = BH, \dfrac{AH}{AD} = \dfrac{HP’}{P’D}$, suy ra $\dfrac{HL}{HB} = \dfrac{HP’}{P’D}$. (2)

Từ (1) và (2) ta có $\dfrac{LE}{EB} = \dfrac{HP’}{P’D}$, suy ra $P’E ||HL||BD$, suy ra $P’E \bot AB$.

Chứng minh tương tự ta có $P’F \bot AC$.

Do đó $AEP’F$ nội tiếp, suy ra $P’ \equiv P$. Hay $D, P, H$ thẳng hàng.

Ví dụ 4. (PTNK 2022) Cho tam giác $A B C$ có trực tâm $H, D$ đối xứng với $H$ qua $A$. $I$ là trung điểm của $C D$, đường tròn $(I)$ đường kính $C D$ cắt $A B$ tại $E, F(E$ thuộc tia $A B)$
a) Chứng $\operatorname{minh} \angle E C D=\angle F C H$ và $A E=A F$.
b) Chứng minh $H$ là trực tâm của $\triangle C E F$.
c) $B H$ cắt $A C$ tại $K$. Chứng minh $E F K H$ nội tiếp và $E F$ là tiếp tuyến chung của $(C K E)$ và $(C K F)$.
d) Chứng minh tiếp tuyến tại $C$ của $(I)$ và tiếp tuyến tại $K$ của $(K E F)$ cắt nhau trên đường thẳng $A B$.

Lời giải. Các câu a, b, c dành cho bạn đọc, ở đây mình trình bày lời giải cho câu d.

Lấy $N$ đối xứng với $K$ qua $A B$.
$$
\angle E N F=\angle E K F=\angle E H F=180^{\circ}-\angle E C F \Rightarrow N \in(I) \text {. }
$$
$A P=A K=A N \Rightarrow \angle K N P=90^{\circ} \Rightarrow N P | B C \Rightarrow E N P F$ là hình thang cân.
$\Rightarrow \angle E C N=\angle F C P \Rightarrow \triangle E C N \backsim \triangle A C F$ và $\triangle E C A \backsim \triangle N C F$.
$\Rightarrow \frac{N E}{A F}=\frac{E C}{A C}$ và $\frac{E A}{N F}=\frac{C A}{C F}$
$\Rightarrow \frac{N E}{E C}=\frac{A F}{A C}=\frac{A E}{A C}=\frac{N F}{C F}$
Tiếp tuyến tại $N$ và $C$ của $(I)$ cắt nhau tại $S, S F$ cắt $(I)$ tại $E^{\prime}\left(E^{\prime} \neq F\right)$
$\triangle S E^{\prime} N \backsim \triangle S N F \Rightarrow \frac{N E^{\prime}}{N F}=\frac{S E^{\prime}}{S N}$
$\triangle S E^{\prime} C \backsim \triangle S C F \Rightarrow \frac{E^{\prime} C}{C F}=\frac{S E^{\prime}}{S C}$
$\Rightarrow \frac{N E^{\prime}}{N F}=\frac{E^{\prime} C}{C F}$
Từ (1) và $(2)$ suy ra: $E \equiv E^{\prime}$
Mà tiếp tuyến tại $N$ của $(I)$ đối xứng với tiếp tuyến tại $K$ của $(E H F)$ qua $A B$ nên ta có đpcm.

Bài tập rèn luyện.

Bài 1. Cho đường tròn $(O)$ và điểm $A$ nằm ngoài $(O)$. Từ $A$ vẽ các tiếp tuyến $AB, AC$ đến $(O)$, một cát tuyến qua $A$ cắt $(O)$ tại $D, E$ sao cho $D$ nằm giữa $A$ và $E$ và tia $AE$ nằm giữa hai tia $AB, AO$. Đường thẳng qua $D$ song song $BE$ cắt $BC$ tại $F$. Gọi $K$ là điểm đối xứng của $B$ qua $E$, chứng minh $A, P, K$ thẳng hàng.

Bài 2. Cho tam giác $ABC$ đều, trên cạnh $AB, AC$ lấy $M,N$ thỏa $\dfrac{AM}{BM} + \dfrac{AN}{CN} = 1$. Chứng minh rằng $MN$ tiếp xúc với một đường tròn cố định.

Bài 3. Cho tam giác $A B C$ có các đường cao $A A_1, B B_1, C C_1$ và trực tâm $H$. Chúng minh rằng đường thẳng Euler của các tam giác $A B_1 C_1, B C_1 A_1, C A_1 B_1$ đồng quy.

Bài 4. (Nga 2017) Cho hình thang cân $ABCD$ có $BC < AD$ và $BC \parallel AD$. Đường tròn $w$ qua $B, C$ cắt cạnh $AB$ tại $X$, đường chéo $BD$ tại $Y$. Tiếp tuyến tại $C$ của $w$ cắt $AD$ tại $Z$. Chứng minh $X, Y, Z$ thẳng hàng.

Sử dụng đánh giá bất đẳng thức để giải hệ phương trình

Một trong các phương pháp khác đặc biệt để giải các hệ phương trình là sử dụng bất đẳng thức, kiểu $A \geq 0$, khi đó $A = 0$ chỉ tại các dấu $=$ xảy ra, hoặc $x \geq y \geq z \geq x$, do đó hệ có nghiệm chỉ khi các dấu $=$ đồng thời xảy ra.

Ta cùng tìm hiểu phương pháp này thông qua một số ví dụ, từ đó rút ra kinh nghiệm giải các hệ phương trình khác.

Ví dụ 1. Giải hệ phương trình

$\left\{\begin{array}{l}
x+\frac{2 x y}{\sqrt[3]{x^2-2 x+9}}=x^2+y \\\\
y+\frac{2 x y}{\sqrt[3]{y^2-2 y+9}}=y^2+x
\end{array}\right.$

Lời giải.

$$
\left\{\begin{array}{l}
x+\frac{2 x y}{\sqrt[3]{x^2-2 x+9}}=x^2+y(1) \\\\
y+\frac{2 x y}{\sqrt[3]{y^2-2 y+9}}=y^2+x(2)
\end{array}\right.
$$
Ta có: $x^2-2 x+9 \geq 8 \Rightarrow \sqrt[3]{x^2-2 x+9} \geq 2 \Rightarrow \frac{2 x y}{\sqrt[3]{x^2-2 x+9}} \leq x y$ Tương tự: $\frac{2 x y}{\sqrt[3]{y^2-2 y+9}} \leq x y$
Do đó: $x+\frac{2 x y}{\sqrt[3]{x^2-2 x+9}}+y+\frac{2 x y}{\sqrt[3]{y^2-2 y+9}} \leq x+y+2 x y \leq x+y+x^2+y^2$
(Dấu “=” xảy ra khi và chỉ khi $x=y$ Từ $(1),(2)$ và $(3)$ suy ra $x=y$ Thay $x=y$ vào (1) ta được:
(4) $\Leftrightarrow \sqrt[3]{x^2-2 x+9}=2 \Leftrightarrow x^2-2 x+9=8 \Leftrightarrow(x-1)^2=0 \Leftrightarrow x=1 \Rightarrow$ $y=1$
Vậy nghiệm của hệ phương trình là $(x, y) \in{(0,0) ;(1,1)}$.

Ví dụ 2. (Hệ hoán vị vòng quanh) Giải hệ phương trình

$$\left\{\begin{array}{l}
x^3+3 x^2+2 x-5=y \\\\
y^3+3 y^2+2 y-5=z \\\\
z^3+3 z^2+2 z-5=x
\end{array}\right.$$

Lời giải. Do vai trò bình đẳng trong hoán vị vòng quanh của $x, y, z$ trong hệ trên, ta có thể giả sử
$$
\begin{aligned}
& x=\max {x ; y ; z} \text {. } \\\\
& \text { Vì } y \leq x \text { nên } x^3+3 x^2+2 x-5 \leq x \\\\
& \Leftrightarrow x^3+3 x^2+x-5 \leq 0 \\\\
& \Leftrightarrow(x-1)\left(x^2+4 x+5\right) \leq 0 \text {. } \\\\
& \text { Vì } x^2+4 x+5=(x+2)^2+1>0 \text { nên } x \leq 1 \text {. } \\\\
& \text { Mà } z \leq x \text { nên } z \leq 1 \text {. } \\\\
& \text { Lập luận ngược lại quá trình trên ta được } \\\\
& (z-1)\left(z^2+4 z+5\right) \leq 0 \\\\
& \Leftrightarrow z^3+3 z^2+2 z-5 \leq z \Leftrightarrow x \leq z \text {. } \\\\
& \text { Do đó } x=z \text {. } \\\\
& \text { Suy ra } x=y=z \text {. } \\\\
& \text { Từ đó ta được phương trình } \\\\
& \quad x^3+3 x^2+2 x-5=x \\\\
& \Leftrightarrow(x-1)\left(x^2+4 x+5\right)=0 \Leftrightarrow x=1 \text {. }
\end{aligned}
$$

Ví dụ 3 (Chuyên Toán PTNK 1997) Tìm tất cả các số dương $x, y, z$ thỏa : $\left\{\begin{array}{l}
\frac{1}{x}+\frac{4}{y}+\frac{9}{z}=3 \\\\
x+y+z \leq 12
\end{array}\right.$

Lời giải.

\begin{aligned}
& \text { Ta có }(x+y+z)\left(\frac{1}{x}+\frac{4}{y}+\frac{9}{z}\right) \leq 36 \Leftrightarrow \frac{y}{x}+\frac{4 x}{y}+\frac{z}{x}+\frac{9 x}{z}+\frac{4 z}{y}+\frac{9 y}{z}-22 \leq 0 \Leftrightarrow \\\\
& \frac{(y-2 x)^2}{x y}+\frac{(z-3 x)^2}{x z}+\frac{(3 y-2 z)^2}{y z} \leq 0 \Leftrightarrow y=2 x, z=2 x, 3 y=2 z \text { Từ đó ta } \\\\
& \text { có } x=2, y=4, z=6
\end{aligned}

Ví dụ 4. (PTNK Chuyên Toán 2103) Giải hệ phương trình $\left\{\begin{array}{l}
3 x^2+2 y+1=2 z(x+2) \\\\
3 y^2+2 z+1=2 x(y+2) \\\\
3 z^2+2 x+1=2 y(z+2)
\end{array}\right.$

Lời giải. Cộng ba phương trình lại ta có:
$3\left(x^2+y^2+z^2\right)+2(x+y+z)+3=2(x y+y z+z x)+4(x+y+z) $

$ \Leftrightarrow 3\left(x^2+y^2+z^2\right)-2(x y+y z+x z)-2(x+y+z)+3=0 $
$\Leftrightarrow(x-y)^2+(y-z)^2+(z-x)^2+(x-1)^2+(y-1)^2+(z-1)^2=0 $
$\Leftrightarrow\left\{\begin{array}{l}
x=1 \\\\
y=1 \\\\
z=1
\end{array}\right.
$
Thử lại thấy $(1,1,1)$ là nghiệm của hệ.

Bài tập rèn luyện

Bài 1. Giải hệ phương trình

$\left\{\begin{array}{l}
x+\dfrac{2 x y}{\sqrt{x^2-2 x+5}}=x^2+y \\\\
y+\dfrac{2 x y}{\sqrt{y^2-2 y+5}}=y^2+x
\end{array}\right.$

Bài 2. Giải hệ phương trình $\left\{\begin{array}{l}
y^{3}-6 x^{2}+12 x-8=0 \\\\
z^{3}-6 y^{2}+12 y-8=0 \\\\
x^{3}-6 z^{2}+12 z-8=0
\end{array}\right.$

Bài 3. Tìm các số không âm $x, y, z$ thỏa
$$
\left\{\begin{aligned}
x y z & =1 \\\\
x^3+y^3+z^3 & =x+y+z
\end{aligned}\right.
$$

Định lý Viete và áp dụng

Định lý 1. (Định lý Viete thuận) Cho phương trình bậc hai $a x^2+b x+c=0$ (a,b, c là các hệ số). Nếu phương trình có nghiệm $x_1, x_2$ thì
$$
S=x_1+x_2=\frac{-b}{a}, \text { và } P=x_1 x_2=\frac{c}{a}
$$
Định lý 2. (Định lý Viete đảo) Nếu có hai số $a, b$ thỏa $a+b=S, a b=P$ thì $a, b$ là nghiệm của phương trình
$$
x^2-S x+P=0
$$

Chú ý: Điều kiện để áp dụng định lý Viete là phương trình bậc hai phải có nghiệm, tức là $\Delta \geq 0$.

Ví dụ 1. Cho phương trình $x^2-2(m+1) x+m=0$
(a) Chứng minh rằng phương trình luôn có 2 nghiệm phân biệt $x_1, x_2$
(b) Tính giá trị các biểu thức sau theo $m$
$$
A=x_1^2+x_2^2+x_1+x_2
$$
(c) Tìm $m$ để $A=18$.
Lời giải. $a=1, b=-2(m+1), b^{\prime}=-(m+1), c=m$
a) Ta có $\Delta^{\prime}=b^{\prime 2}-a c=(-m-1)^2-1 \cdot m=m^2+m+1$.

$\Delta=m^2+m+1=m^2+2 \cdot m \cdot \frac{1}{2}+\frac{1}{4}+\frac{3}{4}=$ $\left(m+\frac{1}{2}\right)^2+\frac{3}{4}>0$ với mọi $m$. Vậy phương trình luôn có hai nghiệm phân biệt $x_1, x_2$.
b) Ta có $A=x_1^2+x_2^2+x_1+x_2$
$=\left(x_1+x_2\right)^2-2 x_1 x_2+x_1+x_2$
$=4(m+1)^2-2 m+2(m+1)$
$=4 m^2+8 m+6$.

c) $A=18 \Leftrightarrow 4 m^2+8 m-12=0 \Leftrightarrow m=$ $1, m=-3$.
Vậy $m$ cần tìm là 1 và -3 .

Ví dụ 2. Tìm $m$ để phương trình $x^2-2(m+1) x+m^2-3=0$ có hai nghiệm phân biệt $x_1, x_2$ thỏa $x_1^2+x_2^2+x_1 x_2=$ $m+7$
Lời giải. $a=1, b=-2 m-2, c=m^2-3$.

Ta có $\Delta^{\prime}=b^2-a c=(m+1)^2-\left(m^2-3\right)=2 m+4$. Phương trình có hai nghiệm phân biệt khi và chỉ khi $\Delta^{\prime}=2 m+4>0 \Leftrightarrow m>-2$.

Theo dịnh lý Viete ta có $x_1+x_2=2(m+1), x_1 x_2=$ $m^2-3$

$x_1^2+x_2^2+x_1 x_2=m+7 \Leftrightarrow\left(x_1+x_2\right)^2-x_1 x_2=m+7$ $\Leftrightarrow 4(m+1)^2-\left(m^2-3\right)=m+7 \Leftrightarrow 3 m^2+7 m=0 \Leftrightarrow$ $m=0(n), m=\frac{-7}{3}(l)$.

Vậy giá trị cần tìm của $m$ là $m=0$.

Ví dụ 3. Cho phương trình $x^2-4 m x+3 m^2+1=0$.
a) Tìm $m$ để phương trình có nghiệm.
b) Gọi $x_1, x_2$ là nghiệm của phương trình, tìm hệ thức độc lập $m$ liên hệ giữa $x_1$ và $x_2$.
Lời giải
a) Ta có $\Delta^{\prime}=4 m^2-\left(3 m^2+1\right)=m^2-1$. Phương trình có nghiệm khi và chỉ khi $\Delta^{\prime} \geq 0 \Leftrightarrow m^2-$ $1 \geq 0 \Leftrightarrow m \leq-1$ hoặc $m \geq 1$.
b) Với điều kiện của a) theo định lý Viete ta có $S=x_1+$ $x_2=4 m(1), P=x_1 x_2=3 m^2+1(2)$.
Từ (1), suy ra $m=\frac{1}{4} S$, thế vào (2) ta có $P=3 m^2+1=$ $\frac{3}{16} S^2+1$.
Hay $x_1 x_2=\frac{3}{16}\left(x_1+x_2\right)^2+1$ là hệ thực liên hệ giữa $x_1, x_2$ độc lập với $m$.

Ví dụ 4. Cho phương trình $x^2-2 m x-2 m-3=0$. Chứng minh rằng phương trình luôn có hai nghiệm phân biệt $x_1, x_2$ và tìm giá trị nhỏ nhất của biểu thức $A=x_1^2+x_2^2-$ $x_1 x_2$.
Lời giải

Ta có $\Delta^{\prime}=m^2+2 m+3$.
Vì $m^2+2 m+3=(m+1)^2+2>0 \forall m$ nên $\Delta^{\prime}>0 \forall m$. Vậy phương trình luôn có hai nghiệm phân biệt với mọi $m$.

Theo định lý Viete ta có $x_1+x_2=2 m, x_1 x_2=-2 m-3$. Khi đó $A=\left(x_1+x_2\right)^2-3 x_1 x_2=(2 m)^2-3(-2 m-3)=$ $4 m^2+6 m+9$.

$A=(2 m)^2+2.2 m \cdot \frac{3}{2}+\frac{9}{4}+\frac{27}{4}=\left(2 m+\frac{3}{2}\right)^2+\frac{27}{4} \geq \frac{27}{4}$. Đẳng thức xảy ra khi $m=\frac{-3}{4}$.

Vậy giá trị nhỏ nhất của $A$ là $\frac{27}{4}$ khi $m=\frac{-3}{4}$.

Bài tập rèn luyện

Bài 1. Cho phương trình $x^2-\sqrt{2} x-\sqrt{3}=0$.
(a) Không giải phương trình, chứng minh phương trình có hai nghiệm $x_1, x_2$.
(b) Tính giá trị của $A=x_1^2+x_2^2-3 x_1 x_2 .(A=2+5 \sqrt{3})$
(c) Tính giá trị của biểu thức $B=\frac{1}{x_1^3-4 x_1 x_2+x_2^3}$
Bài 2. Cho phương trình $x^2-2 m x-1=0$.
(a) Chứng minh rằng phương trình luôn có hai nghiệm phân biệt với mọi $m$
(b) Gọi $x_1, x_2$ là nghiệm của phương trình. Tính $A=$ $x_1^2-3 x_1 x_2+x_2^2$ theo $m$. $\left(A=4 m^2+5\right)$
(c) Tìm $m$ để $A=9 .(m= \pm 1)$
Bài 3. Cho phương trình $x^2-2(m-3) x-2 m+5=0$.
(a) Chứng minh rằng phương trình luôn có nghiệm $x_1, x_2$.
(b) Tìm $m$ để $x_1^2+x_2^2-3 x_1 x_2+x_1+x_2=17$. $\left(m=\frac{3 \pm \sqrt{21}}{2}\right)$

Bài 4. Cho phương trình $x^2-3(m+1) x+9 m^2+2=0$. Tìm $m$ để phương trình có hai nghiệm phân biệt $x_1, x_2$ thỏa $x_1^2+x_2^2-3\left(x_1+x_2\right)+1=0$.
(Không có giá trị $m$ nào thỏa mãn)
Bài 5. Cho phương trình $x^2-3 x-4 m=0$
(a) Tìm $m$ để phương trình có hai nghiệm phân biệt. $\left(m>\frac{-9}{16}\right)$
(b) Tìm $m$ để phương trình có hai nghiệm phân biệt $x_1, x_2$ thỏa $x_1+x_2-x_1 x_2=13\left(m=\frac{5}{2}\right)$
(c) Tính giá trị biểu thức $A=x_1^2+x_2^2-4 x_1 x_2$ theo $\mathrm{m}$ và tìm $\mathrm{m}$ để $\mathrm{A}=14$. $\left(A=9+24 m, m=\frac{5}{24}\right)$
Bài 6. Cho phương trình $x^2-2(m-1) x-1=0$.
(a) Chứng minh rằng phương trình luôn có hai nghiệm phân biệt $x_1, x_2$.
(b) Tìm $\mathrm{m}$ để $x_1^2+x_2^2=5\left(m=\frac{2 \pm \sqrt{3}}{2}\right)$
(c) Tìm giá trị nhỏ nhất của biểu thức $x_1^2+x_2^2+x_1 x_2$ (GTNN là 1 khi và chỉ khi $m=1$ )

Bài 7. Cho phương trình $x^2-2(m+1) x+m=0$
(a) Chứng minh rằng phương trình luôn có hai nghiệm phân biệt $x_1, x_2$
(b) Tìm m để $x_1^2+x_2^2-3 x_1 x_2-x_1-x_2=7$
$$
\left(m=\frac{-5 \pm \sqrt{41}}{8}\right)
$$
(c) Tìm giá trị nhỏ nhất của biểu thức $B=x_1^2+x_2^2$ $\left(B_{\min }=\frac{7}{4}\right.$ khi và chỉ khi $\left.x=\frac{-3}{4}\right)$
Bài 8. Cho phương trình $x^2-2 m x-m-3=0$.
(a) Tìm $m$ dể phương trình có hai nghiệm $x_1, x_2$ thỏa
$$
\begin{aligned}
& \frac{1}{x_1}+\frac{1}{x_2}+\frac{1}{2}=0 \
& \left(m=\frac{-3}{5}\right)
\end{aligned}
$$

(b) Tìm $m$ để phương trình có nghiệm thỏa $x_1^3-x_2^3=$ $10\left(x_1-x_2\right)$ $\left(m=\frac{-1 \pm \sqrt{113}}{8}\right)$
Bài 9. Cho phương trình $(m-1) x^2-2 x+1=0$.
(a) Tìm $m$ để phương trình có hai nghiệm phân biệt. $(m \neq 1, m>2)$
(b) Tìm $m$ để phương trình có hai nghiệm $x_1, x_2$ thỏa $x_1^2+x_2^2+x_1 x_2=3\left(m=\frac{-1}{3}\right)$
Bài 10. Cho phương trình $x^2+2(m+2) x+2 m=0$.
(a) Tìm $m$ để phương trình có hai nghiệm $x_1, x_2$ thỏa $x_1^2 x_2+x_2^2 x_1+x_1+x_2=4$
(không có giá trị $m$ thỏa mãn)
(b) Tìm giá trị lớn nhất của biểu thức $A=x_1 x_2-x_1^2-$ $x_2^2\left(A_{\max }=\frac{-63}{4}\right.$ khi và chỉ khi $\left.m=\frac{-1}{4}\right)$

Đường tròn nội tiếp – Đường tròn bàng tiếp trong tam giác

Định nghĩa và một số tính chất quan trọng

Định nghĩa 1. Đường tròn nội tiếp là đường tròn có tâm là giao điểm ba đường phân giác trong và tiếp xúc với ba cạnh của tam giác.

Định nghĩa 2. Đường tròn bàng tiếp là đường tròn có tâm giao điểm của một phân giác trong và hai phân giác ngoài, tiếp xúc với một cạnh và phần nối dài của hai cạnh còn lại.\\
Trong tam giác có ba đường tròn bàng tiếp ứng với ba đỉnh của tam giác.

Tính chất 1. Cho tam giác $ABC$ nội tiếp đường tròn tâm $O$, đường tròn tâm $I$ bán kính $r$ nội tiếp tam giác tiếp xúc với các cạnh $BC, AC, AB$ tại $D, E, F$.
Gọi $I_a, I_b, I_c$ lần lượt là tâm đường tròn ứng với các đỉnh $A, B, C$. $(I_a)$ tiếp xúc với $BC, AC, AB$ tại $D’,E’, F’$.
Đặt $p = \dfrac{AB+BC+AC}{2}, S = S_{ABC}$.
Ta có một số tính chất sau:
a) $AE = AF = p-a$ và $AE’ = AF’ = p$ và $BD = CD’ = \dfrac{AB+BC-AC}{2}$.
b) $K$ là điểm đối xứng của $D$ qua $I$ thì $A, K, D’$ thẳng hàng.
c) Đường tròn ngoại tiếp tam giác $ABC$ đi qua trung điểm các cạnh của tam giác $I_aI_bI_c$.

Chứng minh.

(a) Ta có $A E=A F, B D=B F, C D=C E$, khi đó $A B+A C-B C=A F+B F+A E+C E-$ $B D-C D=A E+A F=2 A E$, suy ra $A E=\frac{A B+A C-B C}{2}=\frac{A B+B C+A C}{2}-B C=p-a ;$
Ta có $B D^{\prime}=B F, C D^{\prime}=C E$, suy ra $A B+A C+B C=A B+B D^{\prime}+C D^{\prime}+A C=$ $A B+B F^{\prime}+A C+C E^{\prime}=A E^{\prime}+A F^{\prime}=2 A E^{\prime} \Rightarrow A E^{\prime}=A F^{\prime}=\frac{A B+B C+A C}{2}=p ;$

Chứng minh tương tự thì $B D=p-b$ và $C D^{\prime}=C E^{\prime}=A E^{\prime}-A C=p-b$, do đó $B D=C D^{\prime}$.
(b) Ta có $I K=I E, I_a D^{\prime}=I_a E^{\prime}$ nên $\frac{I K}{I_a D^{\prime}}=\frac{I E}{I_a E^{\prime}}$ và $I E / / I_a E^{\prime}$ nên $\frac{I E}{I_a E^{\prime}}=\frac{A I}{A I_a}$; do đó $\frac{A I}{A I_a}=$ $\frac{I K}{I_a D^{\prime}}$, suy ra $\triangle A I K \backsim \triangle A I_a D^{\prime} \Rightarrow \angle I A K=\angle I_a A D^{\prime}$, từ đó $A, K, D^{\prime}$ thẳng hàng.
(c) Ta có $A I_b, A I_a$ là phân giác ngoài và phân giác trong góc $A$ nên $\angle I_a A I_b=90^{\circ}$ hay $I_a A \perp I_b I_c$; chứng minh tương tự ta có $I_b B \perp I_a I_c, I_c C \perp I_a I_b$.

Trong tam giác $I_a I_b I_c$ thì $I_a A, I_b B, I_c C$ là ba đường cao, nên đường tròn ngoại tiếp tam giác $A B C$ chính là đường tròn Euler của tam giác $I_a I_b I_c$ nên đi qua trung điểm 3 cạnh của tam giác này.

Tính chất 2. Cho tam giác $ABC$, đường tròn tâm $I$ nội tiếp tam giác $ABC$ tiếp xúc với $BC, AC, AB$ tại $D, E, F$. Gọi $M, N$ lần lượt là trung điểm $BC, AC$. Khi đó $EF, BI, MN$ đồng quy.

Chứng minh.

Gọi $K$ là giao điểm của $B I$ và $E F$, ta chứng minh $K, M, N$ thẳng hàng.
Ta có $\angle K E C=\angle A E F=90^{\circ}-\frac{1}{2} \angle B A C$ và $\angle K I C=\angle I B C+\angle I C B=\frac{1}{2}(\angle A B C+$ $\angle A C B)=90^{\circ}-\angle B A C$. Suy ra $\angle K E C=$ $\angle K I C$, tứ giác $K E I C$ nội tiếp, do đó $\angle B K C=$ $90^{\circ}$.

Tam giác $K B C$ vuông tại $K$ có $K M$ trung tuyến nên $M K=M B=M C$, suy ra $\angle K M C=$ $2 \angle K B C=\angle A B C$, suy ra $K M / / A B$, mà $M N$ là đường trung bình của tam giác $A B C$ nên $M N / / A B$, do đó $K, M, N$ thẳng hàng.

Tính chất 3. Cho tam giác $A B C$, đường tròn tâm $I$ nội tiếp tam giác $A B C$ tiếp xúc với $B C, A C, A B$ tại $D, E, F . I D$ cắt $E F$ tại $K$, khi đó $A K$ đi qua trung điểm $M$ của $B C$.

Chứng minh. Qua $K$ vẽ đường thẳng song song hay $M$ là trung điểm cạnh $B C$. với $B C$ cắt $A B, A C$ tại $P$ và $Q$, ta chứng minh $K$ là trung điểm $P Q$.

Ta có $\angle I K \perp P Q$, từ đó suy ra $I K P F, I K E Q$ nội tiếp, suy ra $\angle I P K=\angle I F K, \angle I Q K=\angle I E K$ mà $I E F$ cân tại $I$ nên $\angle I E K=\angle I F K$, suy ra $\angle I P Q=\angle I Q K$. Tam giác $I P Q$ cân nên $K$ là trung điểm $P Q$.

Gọi $M$ là giao điểm của $A K$ với $B C$, ta có $\frac{K P}{M B}=$ $\frac{A K}{A M}=\frac{K Q}{M C}$, mà $K P=K Q$ nên $M B=M C$

Tính chất 4. Cho tam giác $A B C$, đường tròn tâm $I$ nội tiếp tam giác $A B C$ tiếp xúc với $B C, A C, A B$ lần lượt tại $D, E, F . E F$ cắt $B C$ tại $P$. Khi đó $\frac{P B}{P C}=\frac{D B}{D C}$ và $I P \perp A D$.
Chứng minh

Theo ví dụ 1.1 ta có $\frac{P B}{P C}=\frac{D B}{D C}$.

Gọi $K$ là giao điểm của $I A$ và $E F$ ta có $\angle I K P=90^{\circ}$, suy ra $I K P D$ nội tiếp, do đó $\angle I P D=\angle I K D$.
Mặt khác $I K \cdot I A=I E^2=I D^2$, suy ra $\triangle I K D \backsim \triangle I D A \Rightarrow I K D=\angle I D A$.
Do đó $\angle I P D=\angle I D A$, suy ra $D A \perp IP$.

Bài tập có lời giải

Bài 1. (PTNK 2014) Cho điểm $\mathrm{C}$ thay đổi trên nửa đường tròn đường kính $A B=2 R$ $(C \neq A, C \neq B)$. Gọi $H$ là hình chiếu vuông góc của $C$ lên $A B ; I$ và $J$ lần lượt là tâm đường tròn nội tiếp các tam giác $A C H$ và $B C H$. Các đường thẳng $C I, C J$ cắt $A B$ tại $M, N$.
(a) Chứng $\operatorname{minh} A N=A C, B M=B C$.
(b) Chứng minh 4 điểm $M, N, I, J$ cùng nằm trên một đường tròn và các đường thẳng $M J, N I$ và $C H$ dồng quy.
(c) Tìm giá trị lớn nhất của $M N$ và giá trị lớn nhất của diện tích tam giác $C M N$ theo $\mathrm{R}$.

Lời giải.

(a) Ta có $\angle H C B=\angle C A B$ (cùng phụ với $\angle A B C$ ) và $\angle H C A=\angle C B A$ (cùng phụ với $\angle B A C$ ).
Ta có $\angle C A N=\angle N A C+\angle A B C=\angle H A N+\angle A C B=\angle C A N$. Suy ra tam giác $C A N$ cân tại $A$ hay $A N=A C$. Chứng minh tương tự ta có $B M=B C$.
(b) Tam giác $C A N$ cân tại $A$ có $A I$ là phân giác nên cũng là trung trực, suy ra $I C=I N$, suy ra $\angle I N C=\angle I C N=\angle I C H+\angle N C H=\frac{1}{2} \angle A C H+\frac{1}{2} \angle B C H=45^{\circ}$.
Tương tự thì $\angle J M C=45^{\circ}$.
Tứ giác $M I J N$ có $\angle J M C=\angle I N C=45^{\circ}$ nên là tứ giác nội tiếp, hay $M, N, I, J$ cùng thuộc một đường tròn.
Tam giác $I N C$ cân có $\angle I C N=45^{\circ}$ nên $\angle C I N=90^{\circ}$, suy ra $C I \perp C M$.
Chứng minh tương tự $M J \perp C N$.
Tam giác $C M N$ có $C H, M J, N I$ là các đường cao nên đồng quy.
(c) Đặt $A C=b, B C=a$. Ta có $a^2+b^2=B C^2=4 R^2$.
Ta có $A N=A C=b, B M=B C=a$.
$A M+B N=B C+M N$, suy ra $M N=a+b-B C=a+b-2 R$.
Ta có $(a+b)^2 \leq 2\left(a^2+b^2\right)=8 R^2$. Suy ra $a+b \leq 2 \sqrt{2} R$, suy ra $a+b-2 R \leq 2 R(\sqrt{2}-1)$. Đẳng thức xảy ra khi $a=b=R \sqrt{2}$.
Vậy giá trị lớn nhất của $M N$ bằng $2 R(\sqrt{2}-1)$ khi $C$ là điểm chính giữa đường tròn. Khi đó $S_{C M N}=\frac{1}{2} C H \cdot M N \leq R^2(\sqrt{2}-1)$. Đẳng thức xảy ra khi $C$ là điểm chính giữa đường tròn.

Bài 2. Cho tam giác $A B C$ có bán kính đường tròn nội tiếp là $r$, đường tròn ngoại tiếp là $R$ và bán kính đường tròn bàng tiếp lả $r_a, r_b, r_c$. Khi đó
$$
r_a+r_b+r_c=4 R+r
$$

Lời giải.

Đường tròn ngoại tiếp tam giác $A B C$ là đường tròn Euler của tam giác $I_a I_b I_c,(A B C)$ cắt $I_b I_c$ tại $N$ và cắt $A I_a$ tại $M$, khi đó $N$ là trung điểm của $I_a I_b$ và $I I_a$. Ta có $M N$ là đường kính của $(A B C)$.
Gọi $K, L$ là hình chiếu của $I_c, I_b$ trên đường thẳng $B C$ và $E$ là hình chiếu của $I_a$ trên $B C$. Tứ giác $I_b L K I_c$ là hình thang vuông có $N P$ là đường trung bình nên $I_c K+I_b L=2 N P$ hay $r_b+r_c+2 N P$. Tương tự $I_a E-I D=2 M P$ hay $r_a-r=2 M P$. Do đó $r_b+r_c+r_a-r=2 N P+2 M P=2 M N=4 R \Rightarrow r_a+r_b+r_c=4 R+r$.

Bài 3. Cho tam giác $A B C$ nhọn có $A B<A C$, đường tròn tâm I nội tiếp tam giác $A B C$ tiếp xúc với các cạnh $B C, A C, A B$ lần lượt tại $D, E, F$. Gọi $K$ là hình chiếu vuông góc với $D$ trên $E F$.
a) Đường tròn ngoại tiếp tam giác $A B C$ và tam giác $A E F$ cắt nhau tại $P$ khác
A. Chứng $\operatorname{minh} P, K, I$ thẳng hàng.
b) $D K$ cắt $A B$ tai $H$. Tính $\angle F P H$.

Lời giải.

a) Chứng minh được $\triangle P F B \backsim \triangle P E C$.
Suy ra $\frac{P F}{P E}=\frac{F B}{E C}$.
Ta cũng chứng minh được: $\angle B K F=\angle C K E$. Hơn nữa $\angle B F K=\angle C E K$ nên $\triangle K F B \backsim \triangle K E C$. Do đó ta suy ra $\frac{F B}{E C}=\frac{K F}{K E}$.
Do vậy $\frac{P F}{P E}=\frac{K F}{K E}$.
Suy ra $P K$ là phân giác góc $\angle E P F$.
Mà $P I$ là phân giác $\angle E P F$ nên $P, I, K$ thẳng hàng.
b) Ta có $H K / / A I$ nên suy ra $\angle P K H=\angle A I P=\angle P F H$.
Do đó tứ giác $P F H K$ nội tiếp.
Suy ra $\angle H P F+\angle H K F=180^{\circ}$.
Mà $\angle H K F=90^{\circ}$ nên $\angle H P F=90^{\circ}$.

Bài tập rèn luyện

Bài 1. (TPHCM 2020) Đường tròn $(I)$ nội tiếp tam giác $A B C$ tiếp xúc với các cạnh $A B, B C$, $C A$ lần lượt tại $D, E, F$. Kẻ đường kính $E J$ của đường tròn $(I)$. Gọi $d$ là đường thẳng qua $A$ song song với $B C$. Đường thẳng $J D$ cắt $d, B C$ lần lượt tại $L, H$.
(a) Chứng minh: $E, F, L$ thẳng hàng.
(b) $J A, J F$ cắt $B C$ lần lượt tại $M, K$. Chứng minh: $M H=M K$.

Bài 2. (TPHCM 2017) Cho tam giác $A B C$ có góc $B$ tù. Đường tròn $(O)$ nội tiếp tam giác $A B C$ tiếp xúc với các cạnh $A B, C A, B C$ lần lượt tại $L, H, J$.
(a) Các tia $B O, C O$ cắt $L H$ lần lượt tại $M, N$. Chứng minh 4 diểm $B, C, M, N$ cùng thuộc một đường tròn.
(b) Gọi $d$ là đường thẳng qua $O$ và vuông góc với $A J ; d$ cắt $A J$ và đường trung trực của cạnh $B C$ lần lượt tại $D$ và $F$. Chứng minh 4 điểm $B, D, F, C$ cùng thuộc một đường tròn.

Bài 3. (PTNK 2015) Cho tam giác $A B C(A B<A C)$ có các góc nhọn, nội tiếp trong đường tròn tâm $O$. Gọi $M$ là trung điểm của cạnh $B C, E$ là điểm chính giữa của cung nhỏ $B C, F$ là điểm đối xứng của $E$ qua $M$.
(a) Chứng minh $E B^2=E F \cdot E O$.
(b) Gọi $D$ là giao điểm của $A E$ và $B C$. Chứng minh các điểm $A, D, O, F$ cùng thuộc một đường tròn.
(c) Gọi $I$ là tâm đường tròn nội tiếp tam giác $A B C$ và $P$ là điểm thay đổi trên đường tròn ngoại tiếp tam giác $I B C$ sao cho $P, O, F$ không thẳng hàng. Chứng minh rằng tiếp tuyến tại $P$ của đường tròn ngoại tiếp tam giác $P O F$ đi qua một điểm cố định.

Hệ phương trình ba ẩn

Trong các bài trước mình đã làm quen với các hệ phương trình hai ẩn, phương pháp chủ yếu cũng là thế, cộng đại số, đặt ẩn phụ. Trong bài này chúng ta tiếp tục với các hệ phương trình nhiều ẩn hơn, chủ yếu là các hệ phương trình ba ẩn, trong các hệ phương trình này có hai dạng ta quan tâm và xuất hiện nhiều là hệ đối xứng và hệ hoán vị vòng quanh.

Hệ ba ẩn đối xứng

Hệ đối xứng ba biến là hệ có dạng

$\left\{\begin{array}{l}
f(x,y,z)=0 \\\\
g(x,y,z)=0 \\\\
h(x,y,z)=0
\end{array}\right.$

trong đó $f, g, h$ là các biểu thức đối xứng với $x, y, z$ tức là khi ta hoán vị $x, y, z$ thì $f, g, h$ vẫn không đổi.

Các biểu thức đối xứng 3 biến cơ bản nhất là $x+y+z, xy+yz+xz, xyz$.

Từ đó ta xét ví dụ sau

Ví dụ 1. Giải hệ phương trình $\left\{\begin{array}{l}
x+y+z=6 (1)\\\\
xy+yz+xz=11 (2)\\\\
xyz=6 (3)
\end{array}\right.$

Lời giải

Từ (1) ta có $y +z = 6-z$, từ (2), $ yz = 11-x(y+z) = 11 – x(6-x) = x^2-6x+11$.

Thế vào (3) ta có $x(x^2-6x+11) = 6$ $\Leftrightarrow x^3 -6x^2+ 11x – 6 = 0$

Giải ra được $x = 1, x = 2, x= 3$.

Với $x = 1$ ta có $y+z = 5, yz = 6$ giải ra được $y = 2, z= 3$ và $y=3, z=2$.

Các trường hợp khác tương tự, hệ phương trình có nghiệm $(1, 2, 3)$ và các hoán vị.

Do đó nếu hệ phương trình ba ẩn đối xứng, có một cách giải là ta tìm được giá trị của các biểu thức đối xứng cơ bản như bài trên.

Ví dụ 2. (PTNK Chuyên toán 2010) Giải hệ phương trình $\left\{\begin{array}{l}
x+y+z=3 \\\\
x y+y z+x z=-1 \\\\
x^3+y^3+z^3+6=3\left(x^2+y^2+z^2\right)
\end{array}\right.$

Lời giải

Ta chỉ cần tính được $xyz$ thì có thể đưa về ví dụ 1.

Từ (1) và (2) ta tính được $x^2+y^2+z^2 = (x+y+z)^2 – 2(xy+yz+xz) = 11$

Suy ra $x^3+y^3+z^3 = 27$

Mà $x^3+y^3+z^3 – 3xyz=(x+y+z)(x^2+y^2+z^2-xy-yz-xz) \Rightarrow xyz = -3$

do đó ta có $x+y+z = 3, xy+yz+xz = -1, xyz = -3$ tương tự ví dụ 1, ta giải được nghiệm là $(1,-1,3)$ và các hoán vị.

Ngoài cách trên ta có thể giải như sau

$x^3+y^3+z^3 = (x+y+z)^3 – 3(x+y)(y+z)(x+z)$, khi đó $(x+y)(y+z)(z+x) = 0$, tổng hai số bằng 0, ta suy ra số còn lại bằng 3, tiếp tục ta cũng có kết quả như trên.

Hệ hoán vị vòng quanh

Các hệ phương trình nhiều ẩn thường gặp là hệ hoán vị vòng quanh có dạng sau:

Phương pháp thường dùng là cộng đại số,phân tích thành tích, sử dụng đánh giá bất đẳng thức để chứng minh $x=y=z$.

Ta xét một số ví dụ sau:

Ví dụ 3. Giải hệ phương trình $\left\{\begin{array}{l}(x-y)^2=2 z-z^2(1) \\\\(y-z)^2=2 x-x^2(2)\\\\ (z-x)^2=2 y-y^2(3)\end{array}\right.$

Lời giải Lấy (1) trừ (2) ta có:

$(x-2 y+z)(x-z)=x^2-z^2-2(x-z)=(x-z)(x+z-2) \Leftrightarrow 2(x-z)(y-1)= 0$

$\Leftrightarrow x=z$ hoặc $y=1$
– $y=1$ ta có $(3) \Leftrightarrow(x-z)^2=1 \Leftrightarrow z=x+1, z=x-1$
+ $z=x+1$ giải được $ x=0, z=1$ và $x=1, z=2 $Khi đó ta có nghiệm $(0,1,1),(1,1,2)$
+ $z=x-1 $ giải ra được $x=1, z=0 $ và $ x=2, z=1 $Ta có nghiệm $(1,1,0)$ và $(2,1,1)$
Với $x=z$ từ (3) ta có $ y^2-2 y=0 \Leftrightarrow y=0, y=2$

Với $y=0$ ta có $\left\{\begin{array}{l}x^2=2 z-z^2 \\\\ z^2=2 x-x^2\end{array} \Leftrightarrow \left\{\begin{array}{l}2 z^2=2 z \\\\ x-z\end{array}\right.\right.$.

Giải được nghiệm $(0,0,0)$ và $(1,0,1)$.

+Với $y=2$, giải ra được nghiệm $(1,2,1)$ và $(2,2,2)$. Vậy hệ phương trình có 8 nghiệm.

Ví dụ 4. (PTNK Chuyên Toán 2103) Giải hệ phương trình $\left\{\begin{array}{l}
3 x^2+2 y+1=2 z(x+2) \\\\
3 y^2+2 z+1=2 x(y+2) \\\\
3 z^2+2 x+1=2 y(z+2)
\end{array}\right.$

Lời giải Cộng ba phương trình lại ta có:
$3\left(x^2+y^2+z^2\right)+2(x+y+z)+3=2(x y+y z+z x)+4(x+y+z) $

$ \Leftrightarrow 3\left(x^2+y^2+z^2\right)-2(x y+y z+x z)-2(x+y+z)+3=0 $
$\Leftrightarrow(x-y)^2+(y-z)^2+(z-x)^2+(x-1)^2+(y-1)^2+(z-1)^2=0 $
$\Leftrightarrow\left\{\begin{array}{l}
x=1 \\\\
y=1 \\\\
z=1
\end{array}\right.
$
Thử lại thấy $(1,1,1)$ là nghiệm của hệ.

Ví dụ 5. Giải hệ phương trình $\left\{\begin{array}{l}
2 x=y^2-z^2 \\\\
2 y=z^2-x^2 \\\\
2 z=x^2-y^2
\end{array}\right.$

Lời giải

Lấy (1) $+(2)$ ta có $(x+y)(x-y+2)=0 \Leftrightarrow x+y=0$ hoặc $x=2-y$.
Với $x+y=0$, từ (3) ta có $z=0$, từ (1) ta có $x=0$ hoặc $x=2$. Ta có nghiệm $(x, y, z)$ là $(0,0,0)$ và $(2,-2,0)$.
Với $x=y-2$, từ (3) ta có $2 z=(y-2)^2-y^2=4-4 y \Leftrightarrow z=2-2 y$. Thế vào (1) ta có: $2(y-2)=y^2-(2-2 y)^2 \Leftrightarrow y^2-2 y=0 \Leftrightarrow y=0, y=2$. Từ đó ta có nghiệm $(-2,0,2)$ và $(2,-2,0)$. Vậy hệ có 4 nghiệm.

Hệ nhiều ẩn không mẫu mực

Một số hệ không mẫu mực thì không có cách giải chung, do đó ta phải để đặc điểm của các hệ phương trình này để có cách giải phù hợp, chủ yếu cũng là giảm được ẩn, phân tích nhân tử, . ..

Ví dụ 6. Giải hệ phương trình sau: $\left\{\begin{array}{l}
(x-2 y)(x-4 z)=55 \\\\
(y-2 z)(y-4 x)=-39 \\\\
(z-2 x)(z-4 y)=-16
\end{array}\right.$

Lời giải

$\left\{\begin{array}{l}(x-2 y)(x-4 z)=55 \\\\ (y-2 z)(y-4 x)=-39 \\\\ (z-2 x)(z-4 y)=-16\end{array} \Leftrightarrow\left\{\begin{array}{l}x^2-2 x y-4 x z+8 y z=55(1) \\\\ y^2-2 y z-4 x y+8 x z=-39(2) \\\\ z^2-2 x z-4 y z+8 x y=-16(3)\end{array}\right.\right.$

Cộng (1),(2),(3) ta có $(x+y+z)^2=0 \Leftrightarrow x+y+z=0 \Leftrightarrow z=-x-y$
Thế vào (1),(2) ta có $\left\{\begin{array}{l}(x-2 y)(5 x+4 y)=55 \\\\ (3 y+2 x)(y-4 x)=-39\end{array}\right.$

$\Leftrightarrow\left\{\begin{array}{l}5 x^2-6 x y-8 y^2=55 \\\\ 3 y^2-10 x y-8 x^2=-39\end{array}\right.$
Nhận thấy $y=0$ không thỏa hpt:
Đặt $x=k y$, ta có hệ

$\left\{\begin{array}{l}
y^2\left(5 k^2-6 k-8\right)=55 \\\\
y^2\left(-8 k^2-10 k+3\right)=-39
\end{array}\right. $
$\Rightarrow-39\left(5 k^2-6 k-8\right)=55\left(-8 k^2-10 k+3\right) $
$\Leftrightarrow 245 k^2+784 k+147=0$
$ \Leftrightarrow\left[\begin{array}{l}
k=-3 \\\\
k=\frac{-1}{5}
\end{array}\right.
$
Với $k=-3$, ta có $y=1$, hoặc $y=-1$. Từ đó ta có nghiệm là $(-3,1,2),(3,-1,-2)$
Với $k=-\frac{1}{5}$ (vô nghiệm)

Chìa khóa trong lời giải này chính là đặc điểm của các hệ số tự do bên phải của các phương trình.

Qua một số ví dụ , hi vọng các em rút ra kinh nghiệm trong việc giải một số hệ phương trình nhiều ẩn, cùng rèn luyện các bài toán sau nhé.

Bài tập rèn luyện

Bài 1. Giải các hệ phương trình sau

1)$\begin{cases} x^2(y+z)^2=(3x^2+x+1)y^2z^2&\\\\y^2(z+x)^2=(4y^2+y+1)z^2x^2&\\\\z^2(x+y)^2=(5z^2+z+1)=x^2y^2 \end{cases}$ 2)$\left\{ \begin{array}{l}xy = x + 3y\\\\yz = 2\left( {y + z} \right)\\\\xz = 3\left( {3z + 2x} \right)\end{array} \right.$ 3) $\left\{ \begin{array}{l}
{\left( {x + y + z} \right)^3} = 12t\\\\
{\left( {y + z + t} \right)^3} = 12x\\\\
{\left( {z + t + z} \right)^3} = 12y\\\\
{\left( {t + x + y} \right)^3} = 12z
\end{array} \right.$

Bài 2. Giải hệ phương trình sau:

1)$\left\{\begin{array}{l}
x^{3}+x^{2}+x-2=y \\\\
y^{3}+y^{2}+y-2=z \\\\
z^{3}+z^{2}+z-2=x
\end{array}\right.$
2) $\left\{\begin{array}{l}
y^{3}-6 x^{2}+12 x-8=0 \\\\
z^{3}-6 y^{2}+12 y-8=0 \\\\
x^{3}-6 z^{2}+12 z-8=0
\end{array}\right.$
Bài 3. Giải hệ phương trình $\begin{cases}ab+c+d=3&\\\\bc+d+a=5&\\\\cd+a+b=2&\\\\da+b+c=6 \end{cases}$

Bài 4.

Cho $a \in \mathbb{R}$. Giải hệ phương trình $\begin{cases} x_1^2+ax_1+(\dfrac{a-1}{2})^2=x_2&\\\\
x_2^2+ax_2+(\dfrac{a-1}{2})^2=x_3&\
…&\\\\
x_n^2+ax_n+(\dfrac{a-1}{2})^2=x_1
\end{cases}$